Gyne All in One

Download as docx, pdf, or txt
Download as docx, pdf, or txt
You are on page 1of 119

Gyne 1 docx

Anatomy of the Female Reproductive System


External genitalia—vulva
 Mons pubis
 Labium majus
 Labium minus
 Clitoris
 Vaginal vestibule
vestibule bulb
major vestibule gland
urethral orifice
vaginal orifice

The blood supply of the external genitalia


comes from the branches of the (a) internal pudendal artery (b)
branches of femoral artery. The venous drainage is to the internal
pudendal vein, long saphenous vein and the vaginal/vesical venous
plexus Nerve supply to the external genitalia

cutaneous branch from the ilioinguinal nerve, genital branch of the


genitor-femoral nerve (L1-L2)

Internal genitalia

 vagina
 uterus
The perimetrium
The myometrium
Endometrium
 oviduct /fallopian tube
 ovary
ligaments of the uterus
 round ligament
 broad ligament
 cardinal ligament
 utero-sacral ligament
Uterus nerve supply (T5-T6- motor, T10-L1-sensory)
The adjacent organs of internal genitalia
A: bladder

SouNok
B: urethra
C: rectum
D: ureters
E: appendix
Note: The endometrium events can be divided into three phases.
 Proliferative phase
 Secretory phase
 Menstrual phase
Types of pelvis:
• gynecoid,
• platypelloid,
• android,
• mixed type

The pelvic floor 3 layered muscle


 Outer layer (4 muscles)
• Bulbo carvenosus muscle
• schiocavernosus
• uperficial transverse perineal muscle
• External sphincter ani
 Middle layer
Urogenital diaphragm
 inner layer
 levator ani muscles: pubococcygeus,

iliococcygeus,ischiococcygeus

The rectouterine pouch is the lowest part of the female

pelvis

Joints of the pelvis (false & true pelvis)


• Sacroiliac joint
• Sacrococygeal joint
• Pubis symphysis

Pelvic ligaments
 Sacrotuberous ligaments

SouNok
 Sacrospinous ligaments (from lateral border of sacrum to ischial
spine)
Physiology Of Reproduction In Women
1. The polar bodies are located in the perivitelline space.

2.Sperm capacitation:the physiochemical change in the sperm by which


it becomes hypermotile and is able to bind and fertilize a secondary
oocyte.

3.Acrosomal reaction:activation of acrosomal enzymes which releases


enzymes that help the sperm to digest the zona pellucida and to enter
the oocyte

4.Zona reaction: prevents polyspermy

5.Ovulation: a process in which a secondary oocyte is released from the


ovary following rupture of a mature graafian follicle and becomes
available for conception

6.Morula: 16-64 cell stage that enters the uterine cavity

7.Implantation: occurs on the 6th day through four stages; apposition,


adhesion, penetration and invasion

8.Decidual reaction: the increased structural and secretory activity of


the endometrium that is brought about in response to progesterone
following implantation.
Three layers of the decidua: superficial spongy layer; spongy layer; thin
basal layer

9.Endometrium: the superficial 2/3 is the stratum functionale shed


during menstruation and supplied by the spiral artery. Deep 3rd stratum
basale, supplied by straight basilar arteries

Three phases of uterine cycle


 Proliferative phase(preovulatory/ follicular):under the influence of
estrogen
 Ovulatory phase
 Secretory phase/luteal phase: the length is constant, at 14days

SouNok
Cyclic change in the breast: estrogen leads to proliferation of the ducts
while progesterone leads to lobular and alveoli maturation

Indicators of ovulation: rise in basal body temperature and LH rise

Different periods of the female life


I. Neonatal period : £ 4 weeks
II. childhood: 4 weeks to age of 12
III. Adolescence or Puberty
IV. sexual maturity: maintains for 30 years
V. peri-menopausal period: begin 40, maintains for 10-20 years

Pre-menopause, menopause (last time of menorrhae), post-


menopause
Vi. senility
Puberty: Puberty is the period when the endocrine and gametogenic
functions of the gonads first develop to the point where reproduction is
possible

Puberty Events
a. Thelarche: the development of breasts.(E,P)
b. Pubarche: the development of axillary and pubic hair.(A)
c. Menarche: the first menstrual period.
d. Adrenarche: an increase in the secretion of adrenal
androgens.

Delayed or absent puberty:


 Definition: considered when menarche has failed to occur by the
age of 17.
 Reasons may be due to abnormalities in:
 Hypothalamus
 pituitary (dwarfing + endocrine abnormalities)
 Idiopathic: In some individuals, puberty is delayed even though
the gonads are present and other endocrine functions are normal.

Menopause
 Conception:Gradual unresponsiveness of the ovaries to

SouNok
gonadotropins with advancing age, decreased negative feedback
of the HPO which leads to its decline in function, so that sexual
cycles and menstruation disappear.
 Age:The menses usually become irregular and cease between the
age of 45 and 55.
 Symptoms:
• The uterus and vagina gradually become atrophic.
• Sensations of warmth spreading from the trunk to the face (hot
flash), night sweats, and various psychic symptoms.

Normal Menstruation (periodic)


 The origin of menstrual blood: Predominantly arterial.
 Composition: tissue debris、 prostaglandins, fibrinolysin (unclot).
 Duration of the menstrual cycle: 1to 8 days (3-5days).
 The amount of lost blood: Less than 80ml (30ml). Menstrual cycle

E.g.: LMP 25 Aug. cycle: 28days


25 + 28 = 53→53- 31days = 22 sep. (next MP)
Ovulation
E.g. LMP 15 Feb. Cycle: 35days, present date 21st Feb Ovulation; 21
+ 15=36 – 29= 7th march

The follicle cells secrete estrogen while the luteal cells secrete estrogen
and progesterone.
Follicles: 7 million (fetal), 2million (at birth), 300-500,000 (puberty).
GnRH release is episodic
Cyclic changes
 uterine cycle
 uterine cervix cycle
 vaginal cycle
 the cycle of breasts  indicators of ovulation

Anovulatory cycle
 the first 12-18months after menarche and onset of menopause
 no corpus luteum is formed→no progesterone
 estrogen continues to cause endometrial growth

SouNok
 the proliferated endometrium becomes thick enough to
slough→bleeding occurs

Hypothalamus pituitary ovarian pathway


 hypothalamus secretes GnRH →stimulates FSH and LH release
from the pituitary
 before ovulation, there is a surge in estrogen but progesterone
remains very low
 after ovulation, the corpus luteum is formed and it secretes
progesterone and estrogen both of which have a surge
 in the entire cycle, estrogen has two surges while progesterone
has one surge
 no pregnancy right before ovulation because of the absence of
progesterone
 both FSH and LH have a surge and LH surge triggers ovulation and
very high FSH surge occurs before ovulation→ stimulates the
follicles

Effects of estrogen

PREGNANCY PHYSIOLOGY
1.Pregnancy:The maternal condition of having a developing fetus in the
body beginning with fertilization and ends with expulsion of the fetus

2.Fertilization:it is the process of fusion of the capacitated sperm with


the mature ovum
 It begins with sperm -egg collision and ends with production of
zygote. (a mononucleated single cell .
 It happens 12 hours after ovulation and last about 24hours.

3.Stages of fertilization
Capacitation: in female reproductive tract, the sperms are enabled to
bind to the zona pellucida receptors.

SouNok
4.Acrosome reaction:rupturing of the acrosome releases the enzymes
that dissolve the zona pellucida enabling the sperm to get in

5.Zona reaction: the zona pellucida rebuilds, preventing other sperms


from going in (destruction of the sperm receptors in the zona pellucida).

6.Implantation occurs through three stages


• Apposition
• adhesion,
• Penetration and Invasion.

7.Essential condition of Implantation


 the disappearance of zona pellucida
 syncytiotrophoblast is derived from the cytotrophoblast
 The synchronized development of blastocyst and endometrium
function is in coordination.
 The pregnant women can produce enough progesterone.

8.Decidual reaction: The increased structural and secretory activity of


the endometrium that is brought about in response to progesterone
following implantation is known as decidual reaction
• Basal decidua: the portion of the decidua in contact with the base
of the blastocyst, where the zygote is implanted. It is the maternal
part of placenta.
• capsular decidua: the thin superficial compact layer covering the
blastocyst
• true decidua: the rest of the decidua lining the uterine cavity
outside the site of implantation

9.
• 16 weeks; the sex is discernible as male or female.
• 20 weeks; Heart tones may often be detected by stethoscope.
Movements have been perceived by the mother. The uterine
fundus is near the level of the umbilicus.
• 40 weeks: The term fetus averages 50 cm in length and 3000 g in
weight. The head has a maximum transverse (biparietal) diameter

SouNok
of 9.5 cm. The average fetus therefore, requires cervical dilatation
of almost 10 cm before it can descend into the vagina.

10. The placenta: fetal surface and maternal surface


 The fetal surface: is covered by the smooth and glistening amnion
with the umbilical cord attached at or near its center. Branches of
the umbilical vessels are visible beneath the amnion as they
radiate from the insertion of the cord
 Maternal surface: showing shaggy look with cotyledons limited by
fissures
 Compositions
• Amnion: fetal surface
• Chorion frodosum: attached to the basal deciduas
• Basal decidua

11. placenta-blood circulation


Fetal and maternal blood in their respective vascular systems, without
communication.
• Maternal: spiral A→intervilli space→ spiral V→maternal material
exchange
• Fetus: umbilical A→villi capillaries→umbilical V→fetus

12.The main functions of the placenta


 1. Metabolic function:Transfer of nutrients and waste products
between the mother and fetus. In this respect, it attributes to the
following functions:
Respiratory; Excretory; Nutritive
 2. Endocrine function: Placenta is an endocrine gland. It produces
both steroid and peptide hormones to maintain pregnancy .
 3. Barrier function.
 4. Immunological function.
13.Fetal membrane (surrounds the fetus): It consists of two layers: outer
chorion and the inner amnion.
Functions:
 Contribute to the formation of liquor amnii;
 Intact membranes prevent ascending uterine infection;

SouNok
 Facilitate dilatation of the cervix during labor;
 Has got enzymatic activities for steroid hormonal metabolism;
 Rich source of glycerophospholipids containing arachidonic acid —
precursor of prostaglandin E2 and F2α
14.At birth, the mature cord is about 30-100 cm in length and 8-20 mm
in diameter

15. The amniotic fluid


 Sources:
• early trimester: the ultra-filtrate of maternal plasma
• After mid trimester :fetal urine is the main source
• fetal lungs, amnion, fetal skin, Wharton's jelly
 Absorption: fetal membrane (50%); digestive canal, umbilical
cord, fetus skin
• The amniotic fluid is
continuously exchanged at a rapid
rate
 Capacity:800ml at term, post-
term pregnancy<300ml.
Status of amniotic fluid: pH: 7.20; Density: 1.007-1.025.
 Contents: water(98-99%), inorganic substance, organic substance
The function of amniotic fluid
1.Protect fetus : It cushions the fetus against severe injury; provides a
medium in which the fetus can move easily ,may be a source of fetal
nutrients;
2.Protect mater: prevent infection
3. a source for analysis of fetal tissues and fluids . the chromosome
disease eg:Down sundrome
16.Maternal changes during pregnancy
 Genital Tract
 Breast
 Cardiovascular System
 Hematologic system
 Pulmonary System
 Gastrointestinal Tract
 The maternal system may produce a series of changes in order to
adapt to the needing of fetal growth and development influenced

SouNok
by placenta hormone and neuroendocrine. Those maternal
adaptions maintain a healthy environment for the fetus.
 most systems return to prepregnancy status within 6 weeks
postpartum.
17. Supine hypotensive syndrome: during late pregnancy, the gravid
uterus produces a compression effect on the inferior vena cava when
the patient is in supine position. This results in hypotension, tachycardia
and syncope. The normal blood pressure is quickly restored by turning
the patient to lateral position
Abortion and ectopic pregnancy
Complications of Abortion
Severe or persistent hemorrhage:
anemia
hypovolemia --- life-threatenina
Sepsis:
develops in neglected care
induced abortion in unsafe place and hands.
Late complications: intrauterine synechiae
infertility
infection involving adnexa and uterus
Classification of Ectopic Pregnancy
• 1. Tubal ( > 95%)
• 2. Other sites ( < 5% )
• 3. heterotopic pregnancy
combination with an intrauterine pregnancy.
• 4. bilateral ectopic : very rare.
Etiology of Ectopic Pregnancy
• Inflammation and infection : PID
• Contraceptive use
• Tubal sterilization
• Tubal surgery
• Prior abdominal surgery
• Others
previous ectopic pregancy / abortion.infertility salpingitis /
exposure to diethylstibestrol
smoking / zygote abnormalities / ovarian factors
Clinical Features-symptoms and sign of ectopic pregnancy
Symptom
• Pain

SouNok
• Bleeding
Spotting
Decidual sloughing
• Amenorrhea
Second amenorrhea is variable.
Spotting at the time of their expected menstruation and thus do
not realize that they are pregnancy.
• Syncope:
dizziness / light headedness / syncope.
It represents advanced stages of intra-abdominal bleeding.
sign
Tenderness :
• Diffuse or localized abdominal pain
• Adnexal tenderness
• Cervical motion tenderness
Adnexal mass :
• A unilateral adnexal mass
Uterine changes
• Typical changes of pregnancy including softening and a slight
increase in size.

SPONTANEOUS ABORTION
It is defined as delivery occurring before the 20th completed week of
gestation. It implies delivery of all or any part of the products of
conception, with or without a fetus weighing less than 500 grams. (The
most common complication of pregnancy)
Etiology
 Morphologic and Genetic Abnormalities (Aneuploidy> 50%)
 Maternal Factor
 Systemic Disease
 Uterine Defect
 Immunologic Disorder  Malnutrition, toxins, trauma

Pathology of Abortion
i. Hemorrhage( into the decidua basalis).
ii. Necrosis and inflammation appear in the area of
implantation. iii. Detachment of product of
conception (partially or entirely).

SouNok
iv. Uterine con¬tractions and dilatation of the cervix
v. Expulsion off all or some of the products of conception

Types of Abortion
i. Threatened abortion: bleeding with or without uterine
contractions, no cervical dilatation and no expulsion of the
products of conception.
ii. Inevitable abortion: bleeding with dilatation of the cervix
without expulsion of the products of conception. with or without
rupture of the membranes
iii. Complete abortion: the expulsion of all of the products of
conception; bleeding with cervical dilatation and closure.
iv. Incomplete abortion: the expulsion of some, but not all of the
products of conception. Generally, bleeding is persistent and is
often severe; prolonged cramps are usually present. The fetus and
placenta are usually passed together <10 weeks' duration. >10
weeks, they may be passed separately with a portion of the
products retained in the uterine cavity.
v. Missed abortion: the embryo or fetus dies and is retained in
utero(no heart motion)
vi. Septic abortion :infection of the uterus and sometimes
surrounding structures
vii. Recurrent abortion: 3 or more consecutive pregnancy losses each
with a fetus weighing <500 g.

viii. Blighted Ovum: a failed development of the embryo only a


gestational sac, with or without a yolk sac

Laboratory test
• Complete Blood Count: Anemia, WBC count, ESR.
• Pregnancy Tests: Falling or abnormally low plasma
levels of β-hCG  Cervical cultures: To determine
pathogens in case of infection.

Complication
o Severe or persistent hemorrhage
o Sepsis, Infection, o Intrauterine

SouNok
synechia, o Infertility o
Perforation 、injury to the bowel
and bladder、hemorrhage,
infection, and fistula formation ( D
and C)

Treatment
a. Threatened abortion: Bed rest and pelvic rest; Prognosis is good
when bleeding and/or cramping resolve.
b. Incomplete abortion: evacuation of the uterus by suction D and C
should be promptly performed; cross-match for possible blood
transfusion and deter-mination of Rh status should be obtained.
The prog¬nosis for the mother is excellent if the retained tissue is
promptly and completely evacuated.
Complete abortion: The patient should be observed for further
c.
bleeding; the products of conception should be examined. The
prognosis for the mother is excellent.
• If abortion occurs after the first trimester: hospitalization, Oxytocic
Ergot, D and C should be administered
d. Septic abortion: Hospitalization, Intravenous antibiotic therapy, D
and C, hysterectomy

ECTOPIC PREGNANCY
A fertilized ovum implants in an area other than the endometrial lining
of the uterus. More than 95% of extrauterine pregnancies occur in the
fallopian tube.

Classification
I. Tubal (> 95%): Includes; ampullary (55%), isthmic (25%), fimbrial
(17%), and interstitial (2%).
II. Others ( < 5% ): Includes cervical , ovarian , and abdominal ( most
abdominal pregnancies are secondary preg¬nancies, from tubal
abortion or rupture and subse¬quent implantation in the bowel,
omentum, or mes¬entery

Etiology
• Tubal Factors: salpingitis

SouNok
• Ovarian Factors
• Other Factor: Intrauterine device ( IUD )
• Bleeding is of uterine origin
Time of rupture

a. Isthmic pregnancies: the earliest, at 6 to 8 weeks due to the small


diameter
b. Ampullary pregnancies: later, at 8-12 weeks.
c. Interstitial pregnancies: the last, at 12-16 weeks as the
myometrium allows more room to grow than the tubal wall.
Interstitial rupture is quite dangerous, as its proximity to uterine
and ovarian vessels can result in massive hemorrhage.

Clinical findings
 Bleeding 75% (spotting decidual sloughing)
 Abdominal Pain 100% (unilateral or bilateral / localized or
generalized / subdiaphragmatic or shoulder pain (intra-abdominal
bleeding)
 Amenorrhea 50%
 Syncope (30-50%), Dizziness, lightheadedness
 Decidual Cast

Signs
 Tenderness
 Adnexal Mass
 Uterine Changes

Culdocentesis: the transvaginal passage of a needle into the posterior


cul-de-sac in order to determine whether free blood is present in the
abdomen.

Treatment:
A. Expectant Management: when β-hCG titer is low ( < 200 mlU/mL )
or decreasing, and the risk of rupture is low
B. Surgical Treatment
C. Emergency Treatment: Immediate surgery is indicated when the
diagnosis of ectopic pregnancy with hemorrhage is made.

SouNok
D. Medical Management: small, unruptured ectopic pregnancies in
asymptomatic woman.

Pregnancy With Internal Medical Diseases

Complication of Heart Disease in Pregnancy


 heart failure: 32-34weeks, delivery stage, puerperium(3d)
 Subacute infective endocarditis
 Anoxia and cyanosis
 venous embolism and pulmonary embolism
Preconception counseling

★ 1 、preconception counseling

suitable to pregnancy not suitable to pregnancy

 the damage of heart  the damage of heart


function is slightly function is seriously
 grade Ⅰ、Ⅱ  grade Ⅲ 、Ⅳ
 without heart failure in the  congestive heart failure
past history
 without complications  serious cardiac arrhythmias,
pulmonary hypertension
 age> 35

The diagnosis of early heart failure


 less than ordinary activity causes discomfort
: palpitation, chest distress, short breath.
 Heart rate exceeds 110 per minute and breath rate exceeds 20 per
minute at rest.
 orthopnea or sit up to breath fresh air because of chest distress at
night.
 Auscultation a few moist rales appear and persistent at base of
lung, cannot vanish after coughing.

SouNok
Female,27 years old,Atrial septal defect :1cm
Feel discomfort only after ordinary activity.

 Can I have a baby? → Yes


 What is the risk for me and Heart failure、 fetal demise、
my baby? → congenital heart disease

What should I do during the



coure of pregnancy? → Intensive care and diagnosis of
early heart failure
 By which way should I
delivery my baby? → Vaginal delivery
 Any special thing to be paid Prevent infection and
attention to after birth? → postpartum hemorrhage

MCQ
Body or corpus ; Cervix;
Before puberty: Corpus: cervix 1:2
At puberty : Corpus: cervix 2:1
Oviduct
1) Interstitial portion: 1cm,the narrowest part
2) Isthmic portion: 3-4cm,narrow
3) Ampulla portion: 5-8cm,wide and tortuous. fertilization
4) Fimbria portion: 1-1.5cm,funnel-shaped mouth
Posterior Pituitary hormones
 Oxytocin
 Arginine vasopressin
the anterior pituitary
 (FSH LH TSH ACTH GH Prolactin)
 FSH and LH are responsible for ovarian follicule stimulation .
 Half-life : LH: 20 minute, FSH: 40 minutes
implantation
 Implantation occurs in the endometrium on the 6th-8th day after
fertilization.
 Implantation occurs through three stages : apposition, adhesion,
and invasion.
-The endometrium events can be divided into three phases.

SouNok
 Proliferative phase
 Secretory phase
 Menstrual phase
During priliferative phase,endometrium grows from approximately
0.5mm to 3-5mm in height.

Fetal growth and development


 During the first 8th weeks, all the major organs are formed.
 After the eighth week, further growth and organ maturation
occur.
 8 weeks; The organs are beginning to form.
 12 weeks; The fingers and toes have nails, and the external
genitalia may be recognizable.
 16 weeks; The sex is discernible as male or female.
 20 weeks; Heart tones may often be detected by stethoscope.
Movements have been perceived by the mother. The uterine
fundus is near the level of the umbilicus.
 24 weeks; Some fat is beginning to be deposited beneath the
wrinkled skin. Viability is reached by the 24th week, but survival at
this stage is still relatively rare.
 28 weeks; The lungs are now capable of breathing, but the
surfactant content is low;
 32 weeks: If born at this stage, 50%-60% infants survive.
 36 weeks: The skin has lost its wrinkled appearance. The chances
for survival are good
 40 weeks: The term fetus averages 50 cm in length and 3000g in
weight. The head has a maximal transverse (biparietal) diameter
of 9. 5 cm. The average fetus, therefore, requires cervical
dilatation of almost 10 cm before it can descend into the vagina.
Umbilical cord
 At birth, the mature cord is about 30-100 cm in length and 8-20
mm in diameter.
 the amniotic fluid : pH: 7.20; Density: 1.007-1.025.
Fetal skull:
1.Sagittal suture : 2.Frontal suture:
3.Coronal suture: 4Lambdoidal suture:
• Active labor: 3 or 4 stages
Tubal rupture
Rupture time
• isthmic rupture : at 6-8 weeks

SouNok
• ampullary rupture : at 8-12 weeks
• interstitial rupture : about 4 months
Sonography: the most important diagnoses method—simplest, precise,
safest
• attention: 28weeks
• <28W: the temporary state of placenta previa, should follow
up.

Gyne 2 docx

Antepartum hemorrhage
PLACENTA PREVIA:it is defined as the abnormal implantation of the
placenta over the internal cervical os.
When gestation is over 28 weeks, the placenta attaches to the low
segment of uterus,partially or completely covering the internal os of
cervix, the position below the fetal presention. previa denotes the
position of the placenta in relation to the presenting part.
Abnormalities of placenta
 Bilobed placenta
 Succenturiate placenta
 Horseshoe placenta
 Lingual –shaped placenta
 Long and round placenta
Please write the classification of the Placenta Previa.
1. Complete or central placenta previa
2. Partial placenta previa
3. Marginal placenta previa
4. Low lying placenta: Placenta is in lower segment, but the lower
edge does not reach internal os
Etiology of placenta previa

 Endometrial factors:
• Scarred or poorly vascularized endometrium in the uterine corpus.
• Endometritis: Puerperal
infection、prolificacy、curettage、cesarean section et al®
endometritis, the trauma of endometrium® to take more nutrition
,placental area enlarged.

SouNok
 Placental factors:
• Large placenta( multiple pregnancy)
• Abnormalities of placenta (Bilobed placenta, Succenturiate
placenta, Horseshoe placenta, Lingual –shaped placenta, Long and
round placenta)
 Zygote factors: the retardation of zygote growth.

Please write the most characteristic symptom of the Placenta Previa.


 ①Painless hemorrhage.
 ②Time: late pregnancy (after the 28th week) and delivery
 ③Characteristics: sudden, painless and profuse
Clinical manifestations
 Symptoms:
 painless hemorrhage
 Character: sudden onset, painless, apparently causeless and
recurrent
 Time: late pregnancy (after the 28th week) and delivery
Unrelated to activity, often occurs during sleep, in a pool of blood.
 Why?
 The low segment of the uterus prolonged, disappearance of
the cervical canal as well as cervical dilatation, the placenta will
not extend, leading to the rupture of blood sinus.
 Signs:
 A
n
e
m
i
a
a
n
d
s
h
o
c
k

SouNok
 A
b
d
o
m
i
n
a
l
e
x
a
m
 T
h
e
si
z
e
o
f
u
t
e
r
u
s
is
p
r
o
p
o
r
ti
o
n

SouNok
a
t
e
t
o
t
h
e
w
e
e
k
o
f
g
e
s
t
a
ti
o
n
 the uterus is soft, relaxed, elastic, no
tenderness
 fetal mal-presentation and fetal distress
 High floating of presenting part
 Hear the placental souffle above the pubic symphysis

Complications
 Maternal:
• Postpartum hemorrhage, shock, death
• Puerperal infection
• Placenta increta: myometrium penetrated by trophoblast of
placenta Thus, the placenta grows completely through the
endometrium.
 Fetus:

SouNok
• Preterm delivery
• Perinatal mortality rate is increased

High risk factor:


• repeated abortion and curretage,
• the history of cesarean section,
• prolificacy,
• multiple pregnancy

Diagnosis

i. Clinical symptoms and signs


ii. Sonography: the most important method-simple, precise, safe
(≥28weeks). <28W, the uterine size is small and placenta
occupies most volume (observation should be done)
iii. MRI
iv. Diagnoses in parturition
• The shape of placenta
• The distance from edge of placenta to the rupture of the
fetal membranes is less than 7cm
• Clot in the maternal side of placenta
Differential diagnosis
I. Placental abruption
II. Rupture of the marginal sinus
III. Rupture of the vasa previa in cord velamentous insertion IV.
Bleeding of cervix and vagina

Treatment
I. Expectant therapy (should not exceed the 36th week)
Why (preterm, prematurity)
 Indications
• <34 week
• Fetal weight <2000g
• Bleeding not severe
II. Termination of pregnancy
• Severe hemorrhage

SouNok
• >36weeks GA
• Mature fetal lungs
Fetal death/distress (34-36wks)
When is the time limit of the Expectant therapy in the Placenta Previa?
Time limit: to 36w
 After 36 weeks , the benefits of additional maturity must be
weighed against the risk of major hemorrhage.
 The possibility that repeated small hemorrhages may be
accompanied by intrauterine growth retardation must also be
considered.
 About 75% cases of placenta previa are now terminated at
between 36 and 40 weeks.
Indication of Termination of pregnancy (Delivery )
 hemorrhage is so severe as to stop the pregnancy despite the
immaturity of the fetus
 more than 36 weeks gestation
 The lung of fetus is mature
 34W-36W, fetal distress, after promoting fetal lung maturity
 fetal dead
Cesarean Section of placenta previa
Indications:
• ① degreeⅡ,especially nulliparity,can not end the delivery in
a short time.
• ② degreeⅠ,together with fetal distress, to rescue the fetus;
• ③ degreeⅢ,the patient's condition deteriorated,fetal
death,can not end the delivery in a short time.
• ④ there is no progress in labor after artificial rupture of fetal
membrane
What should be notice at Cesarean Section when choosing of
incision ?
(1)Cesarean section
 The main and first method
ÓIndications:
 central placenta previa.
 Partial placenta previa and marginal placenta previa with severe
bleeding, who could not vaginal delivery as soon as possible.
 fetal distress and fetal malpresentation
ÓIncision of uterus: To avoid the placenta as possibly, vertical or low
transverse.

SouNok
PLACENTAL ABRUPTION:after 20 weeks of gestation or during the
course of delivery, the separation of the placenta from its site of
implantation before the baby is delivered resulting in hemorrhage→fetal
distress/death.
Placental abruption is the premature separation of the normally
implanted placenta from the uterine wall, resulting in hemorrhage
between the uterine wall and the placenta.

Types
i. the revealed abruption ( the external bleeding )
ii. The concealed abruption ( the internal bleeding )
iii. mixed bleeding

Etiology
 Local vascular injury (Hypertensive Disorders Complicating
Pregnancy, chronic hypertension, chronic renal disease)
 Sudden decrease in the pressure of uterus
 Trauma, short cord syndrome

Clinical findings
 Abdominal pain with/without vaginal bleeding
 Degrees
• Mostly occur during delivery. <1/3 abruption
• In the third trimester of pregnancy. ½ abruption(uterus >GA
due to hematoma)
 Abruption >1/2
 Internal hemorrhage
 Persistent abdominal pain
 Shock
 Mostly caused by accident or pre-eclampsia

What is the most important Complication in Placental abruption?


 fetal death
 DIC
 postpartum hemorrhage
 Acute renal failure
 Amnionic fluid embolism

SouNok
Differential diagnosis

vi

a

SouNok
t

Treatment of Placental abruption


Principle: diagnosis timely , Once make the diagnosis of placental
abruption, Termination of pregnancy timely.
★ Treatment of hypovolemic shock:intensive transfusion with
blood
★ Termination of pregnancy timely: Vaginal delivery, CS
★ treatment of complications
-- postpartum hemorrhage
-- coagulation dysfunction 、DIC
-- acute renal failure
High risk factors of Placental abruption
 Increased age and multiparity
 uterine distention (multiple pregnancy)
 vascular deficiency or deterioration ( diabetes mellitus)
 uterine anomalies or tumors ( leiomyoma)
 cigarette smoking, alcohol consumption
 Blood clotting disorders (thrombophilia)
HYPERTENSIVE STATE IN
PREGNANCY

SouNok
Hypertensive states in pregnancy
 Gestational hy¬pertension (pregnancy-induced hypertension)
 preeclampsia (mild and severe)
 eclampsia
 chronic hypertension ( either essential, or secondary to renal
disease, endocrine disease, or other causes )
 chronic hypertension with superimposed preeclampsia

Hypothesis
An immunologic dis¬turbance causes abnormal placental implantation
→decreased placental perfusion→ stimulates the production of
substances in the blood →activate or injure endothelial cells→The
vascular endothelium provides a single target for these blood-borne
products, →the mul¬tiple organ system involvement in preeclampsia

Pathophysiology
a. Central Nervous System: headache, convulsion, coma
b. Eyes: retinal edema, detachment, blindness
c. Pulmonary System
d. Cardiovascular System: HF/ pulmonary edema
e. Kidney: injury and swelling of the endothelial cells; proteinuria;
oligura
f. Liver: HELLP SYNDROME
g. Blood : DIC
h. Placenta-fetus

Gestational hypertension
a. Hypertension: Occurs 20weeks after gestation and recover 12
weeks postpartum. SP≥140mmHg and
(or)DP≥90mmHg
b. proteinuria(-) c.Diagnosed only after delivery

Preeclampsia
 Mild-preeclampsia
 Hypertension : ≥20 gestational weeks; SP≥140mmHg and
(or)DP≥90mmHg
 Proteinuria: urine protein ≥0.3g/24h, urine protein (+)

SouNok
Preeclampsia
 Hypertension + Proteinuria(-) + any of the following :
① Thrombocytopenia: (platelet count < 100 ×109/L);
② Renal insufficiency :serum creatinine ≥ 1.1mg/ dl or a doubling of
the serum creatinine concentration in the absence of other renal
disease;
③ Impaired liver function : ALT、 AST ↑
④ Pulmonary edema
⑤ New-onset cerebral or visual disturbances
Complication of preeclampsia
Fetal risks :
• intrapartum fetal distress or stillbirth.
• intrauterine growth restriction
Maternal risks:
• Eclampsia
• Cerebrovascular accidents
• Abruptio placentae
• HELLP syndrome
Severe preeclampsia
a. BP:SP ≥160mmHg and (or)DP≥110mmHg
b. severe headache or visual changes
c. Heart failure or pulmonary edema.
d. epigastric pain; (RUQ pain)
e. elevation of transaminases : ALT、 AST ↑
f. proteinuria ≥ 2 g/ 24h or urine protein ≥(+++)
g. abdominal dropsy or pleural effusion
h. acute renal failure with rising creatinine
i. oliguria : < 400 ml/24 h or <17ml/h
j. Thrombocytopenia (platelet count < 100 ×109/L) 、intravascular
hemolysis, anemia、jaundice、LDH↑. k. before 34 gestational weeks.

Expectant Management of severe preeclampsia


1. Bed rest: left-lateral position
2. Daily weight measurement
3. Anti-spasm treatment: MgSO4
4. Anti-hypertensive treatment
5. Steroid :to accelerate lung.
<35w,may delivery in 1w.dexamethasone ,6mg bid, im

SouNok
×2d.
6. Termination of Delivery
Attention of using magnesium sulpate (MgSO4)
① knee reflex(+)
② R ≥ 12bpm ;
③ urine ≥17ml/h or ≥400ml/24h
④ Mgso4 concentation monitoring
⑤ prepare calcium gluconate
⑥ lower dose or stop use when renal dysfunction
 If something happens…
①stop MgSO4
② 10% calcium gluconate 10 ml iv ,5-10min
Termination of Delivery ※indications for preeclampsia
• Bp ≥160/100 despite treatment.
• Urine output<400ml/24 hours.
• Platelet count<50 x 109/l
• Progressive increase in serum creatinine.
• LDH > 1000IU/L.
• NST show Repetitive late deceleration with poor variability
• Severe IUGR with oligohydramnios.
• Decreased fetal movement.
• Reversed umbilical diastolic blood flow
Eclampsia:preeclampsia + convulsions and or unexplained coma during
pregnancy or postpartum .
Management of Eclampsia
 General measures: airway,Oxygen
 Control of convulsion: first line medicine- MgSO4 超链接
 Control hypertension:
 Delivery:the definitive treatment
Eclampsia
A. preeclampsia + seizures
B. Occurrence: prenatal 、intrapartum、 postpartum
 the convulsive movements : suddenly,1-1.5min, Without
breathing
• Facial twitchings
• generalized muscular contraction
• jaws close violently
• all muscles alternately contract and relax
• lies motionless

SouNok
Chronic hypertension and pregnancy
 Chronic hypertension: SP≥140mmHg and(or)DP≥90mmHg
before pregnancy or before 20 weeks' gestation persists for more
than 12 weeks postpartum.
 proteinuria(-)

Chronic hypertension superimposed on preeclampsia


i. chronic hypertension + proteinuria
ii. SP≥140mmHg and(or)DP≥90mmHg, before pregnancy or
before 20 weeks' gestation, persists for more than 12 weeks
postpartum.
iii. chronic hypertension + higher BP iv. thrombocytopenia (platelet
count < 100 ×109/L);

Treatment
 Individualized treatment
 Gestational hypertension: Rest, sedation, close monitoring of
mother and infant, Anti-hypertension according to the indication.
 Preeclampsia: sedation, antispasm, anti-hypertension according to
the indication, monitoring maternal and infant, terminate
pregnancy timely.
 eclampsia : control the seizures , terminate pregnancy timely

Treatment
I. Assessment monitoring: BP, assistant examination,
NST
II. Common treatment: left lateral position, diet;
enough protein calories III. Antispasm: MgSo4
IV.
Se
da
ti
o
n:
di
az
ep

SouNok
a
m
V.
A
n
ti
hy
pe
rt
en
si
ve
VI. Diuretics
VII. Promote fetal lung maturation VIII.
Terminate pregnancy

Eclampsia
 Control seizure: first line medicine- MgSO4、 hibernate mixture
 Correct acidosis and hypoxia:4% NaHCO3
 Control hypertension
 Terminate pregnancy 2 hours after controlling seizure

Post-partum
 Prevent postpartum eclampsia: MgSO4, 24-48h after delivery.
 Monitoring BP and proteinuria 3-6d after delivery.
 Monitoring and record the hemorrhage

DIABETES MELLITUS
A clinical syndrome characterized by deficiency of or insensitivity
to insulin and exposure of organs to chronic hyperglycemia,is the
most common medical complication of pregnancy
2 Types
I. DM (diabetes
mellitus prior to
pregnancy) II.
GDM( gestational

SouNok
diabetes mellitus)
Diagnostic criteria
 There are three ways to diagnose preexisting dia¬betes mellitus
and each way must be confirmed by a follow up test. Criteria for
diagnosing diabetes melli¬tus include:
 Symptoms of diabetes (polyuria, polydipsia, and/ or unexplained
weight loss) plus a casual plas¬ma glucose concentration ≥200
mg/dL.
 Fasting plasma glucose (at least 8 hours without eating) ≥126
mg/dL.
 Two-hour plasma glucose ≥ 200 mg/dL after drinking a 75 -gram
glucose load.

GDM
Fasting glucose ≥ 5.1mmol/l ,1-hour ≥ 10 mmol/l,2-hour ≥ 8.5mmol/l
The diagnosis of GDM would be made with one out of the three values
elevated

Management of GDM
 Antepartum Management
Preconception counseling of PGDM
Glycemic control during pregnancy
prenatal care
 Intrapartum Management
The optimum time and method of delivery(when?
how?)
Management in labour
 Postpartum Management
 Care of the baby
High risk factors Diabetes Mellitus and Pregnancy(GDM)
 a ) Obesity (nonpregnant body mass index ≥30)
 b) Prior history of GDM
 c) Heavy glycosuria (>2++)
 d) Unexplained stillbirth, prior infant with congenital
malformation e) Family history of diabetes in first degree rela
tion
 f ) Previous macrosomic infant(>4000 gg)
 g ) History of recurrent pre-eclampsia

SouNok
 h ) History of recurrent moniliasis
 i) Maternal age over 30

Effects of diabetes on pregnancy


Depending on the severity of diabetes and blood glucose levels.
Maternal Fetal Neonatal
Maternal:
During pregnancy
1、Abortion:15-30%.
2、Increased incidence of preeclampsia(25%)
3、Infection: Urinary tract infection and vulvovaginitis
4、Polyhydramnios
5、Ketoacidosis
6、Preterm labor:(26%) may be due to infection or polyhydramnios.
Fetal:
1.Fetal macrosomia
2.Congenital malformation
3. Fetal growth restriction
Neonatal
① Hypoglycemia: due to hyperinsulinemia.
② Respiratory distress syndrome
Complications

A. Maternal
 Preeclampsia
 Postpartum hemorrhage
 Ketoacidosis,
 Infection, Diabetic coma
B. Fetal
 Shoulder dystocia
 Abortion, intrauterine death
 Congenital anomalies
 Macrosomia,
 cardiomyopathy

Treatment
 Antepartum care: diet and exercise.
 Insulin is added as needed for glucose control only after clear
dietary errors are noted and attempts at correction are done.

SouNok
 Assessment of the fetus by glucose memory meters combined
with clinical/ultrasound assessment of fe¬tal growth cannot be
replaced by other antenatal tests. (fetal macrosomia,
polyhydramnios-high risk )

Indications for early delivery include


I.

nonreass
uring
fetal
testing
II.
Poor
glycemic
control.
III. worsening of uncontrolled hypertension,
IV. worsening renal disease, poor fetal growth
MEDICAL ILLNESS IN NORMAL
PREGNANCY
Heart Disease in Pregnancy (the second leading cause of maternal
death)
During the 32nd-34th gestational weeks, the blood volume increases to
maximum (30-40 %↑)
• the burden of the heart is heaviest during delivery, the first stage
is caused by uterine contraction (blood enters the systemic
circulation from the uterus)
• second stage when the breath is held: peripheral resistance ↑
• Third stage after delivery of the placenta, the blood volume
increases suddenly.

Cardiovascular changes in normal pregnancy


 gestational period
 During delivery
 Puerperium

SouNok
Returned blood volume increased in 3 days in postpartum and return to
normal after 2~6 weeks.

Classification of heart disease


I. Congenital heart disease
• Left-to-right shunt: atrial septal defect, ventricular septal defect,
patent ductus arteriosus.
• Right-to-left shunt
• Splitless : pulmonary stenosis, aortic stenosis, marfan syndrome
III.Rheu
matic
heart
disease

rhe
um
atic
val
vul
ar
lesi
on
• Mitral stenosis: the most common type.
• Pulmonary congestion, pulmonary edema.
Mild - tolerate pregnancy
Severe-surgery before pregnancy or terminate earlier
 mitral insufficiency: tolerate
 Aortic stenosis:surgery before pregnancy
 Aortic insufficiency:tolerate pregnancy
 The heart disease of hypertensive
disorder
 Peripartum cardiomyopathy (PPCM)

 Myocarditis: after viral infection.

SouNok
The influence on pregnancy
I. Incidence of Abortion,preterm delivery,FGR,fetal
distress,neonatal asphyxia,fetal death raised
II. increased rate of cesarean-section
III. toxic reaction of drug on fetus
IV. inherited congenital heart disease(ventricular septal
defect,Marfan's syndrome)

Complications
a. heart failure: 32-34weeks, delivery stage, puerperium(3d)
b. Subacute infective endocarditis
c. Anoxia and cyanosis
d. venous embolism and pulmonary embolism
NYHA Counselling
 Grade 1: uncompromised & no limitation on physical activity.
 Grade 2: slightly compromised/ slight physical activity limitation
 Grade 3: markedly compromised/ discomfort with less than
ordinary activities
 Grade 4: severely compromised with discomfort even at rest

Pre-conception counseling
 Suitable for pregnancy
• If the damage to heart function is slight
• grade Ⅰ、Ⅱ
• without heart failure in the past
• without complications
 Not suitable for pregnancy
• the damage of heart function is serious
• grade Ⅲ 、Ⅳ
• congestive heart failure history
• serious cardiac arrhythmias,
• pulmonary hypertension
• age >35

SouNok
The diagnosis of early heart failure
 Less than ordinary activity causes discomfort: palpitation, chest
distress, short breath.
 Heart rate > 110/minute and breath rate > 20/minute at rest.
 Orthopnea.
 Auscultation-a few moist rales appear and persistent at base of
lung, cannot vanish after coughing.

ANEMIA IN PREGNANCY
Gestational anemia:Hb <110g/L, Hct <0.33 caused mostly by
hemodilution and Iron deficiency. Mild: Hb> 60g/L; severe: Hb ≤ 60g/L
The effect of anemia on pregnancy
 Effects on mother:
 poor tolerance to surgery、childbirth and anesthesia
 maternal mortality rate ↑(anemic heart disease,
preeclampsia, placenta abruptio)
 Puerperium infection rate ↑
 Effects on baby:
 Amount of iron transferred to the fetus is unaffected even if the
mother suffers from iron deficiency anemia. So the neonate does
not suffer from anemia at birth.
 lack of folate→fetal neural tube defects
 FGR、preterm labor 、Fetal distress even dead

Treatment of IDA
I. Supplementary iron therapy: oral + Vit C (aids absorption)
II. blood transfusion: Hb≤60g/L
III. Intrapartum and postpartum treatment
• preventing postpartum hemorrhage
• preventing infection

Clinical manifestation of diagnosis


 The onset is usually insidious and is first revealed in the last
trimester or may be acutely manifested in early puerperium
 Anemia:Pallor of varying degree,fatigue、dizziness、short

SouNok
breath。
 symptom of digestive tract:loss of appetite、nauseas and
vomit, Anorexia or protracted vomiting ,Occasional diarrhea,
ulceration in the mouth and tongue
 symptom of peripheral neuritis:hand and foot numbness,
tingling (Vit B12)
 others:unexplained fever,enlarged liver and spleen

PREGNANCY WITH SURGICAL


DISEASES
Pregnancy surgical diseases
 Acute appendicitis: Acute appendicitis is the most common
extrauterine complication of pregnancy for which laparotomy is
performed. most cases occur in the second and third trimesters
 Acute cholesystitis and cholelithiasis, Acute intestinal obstruction
Symptoms of acute appendicitis
 Atypical, Right lower- or middle-quadrant pain; (vague pain )
 Rectal and vaginal tenderness are present in 80% of patients,
particularly in early pregnancy.
 Nausea, vomiting, and anorexia are usually present, as in the non-
pregnant patient.

Signs
 Tenderness, Upward displacement of the appendix.
 After the first trimester, the appendix is gradually displaced above
McBurney's point, with horizontal rotation of its base. The
migration continues until the eighth month of gestation.

Differential diagnosis
 Pyelonephritis (the most common misdiagnosis)
 ruptur cholecystitis ed corpus luteum cyst, adnexal torsion,
ectopic pregnancy
 abruptio placentae, early labor, round ligament syndrome
 chorioamnionitis, degenerating myoma, salpingitis, cholangitis

SouNok
Symptoms of acute cholecystitis
• Biliary colic attacks are often of acute onset, seemingly triggered by
meals, and may last from a few minutes to several hours.

• anorexia, nausea, vomiting, dyspepsia , and intolerance of certain


foods, particularly those with high fat content ( nonpregnant state)

Signs
I. Fever, Right upper-quadrant pain, Murphy's sign (tenderness under
the liver with deep inspiration)

Differential Diagnosis
a. Appendicitis
b. symptoms of digestive track
c. pain, WBC ↑
d. AST, ALT, bilirubin and WBC ↑
e. Severe preeclampsia with associated right upper-quadrant
abdominal pain and abnormal liver function tests.
f. proteinuria, nondependent edema, hypertension

Causes of acute intestinal obstruction


 Adhesion 60%
 Volvulus 25%
 Others 15%
Treatment of acute intestinal obstruction
 The same management as non-pregnant patients
 The cornerstones of therapy a. Bowel decompression
b. IV hydration
 Timely surgery
NORMAL LABOUR

Labor is a sequence of uterine contractions that result in effacement and


dilatation of the cervix and voluntary bearing-down efforts leading to
the expulsion per vagina of the products of conception.
Cause of Onset of Labor

SouNok
 Estrogen ↑
 Oxytocin ↑
 Progesterone ↓
 Prostaglandins ↑
Four determinate factors of labour
 Powers,
 Passage,
 Passenger,
 Psychologic
The soft birth canal(mcq)
• lower segment
• cervix
• vagina
• pelvic floor and perineum
Stages of Labor
 The first stage:
---onset of true labor to full cervical dilation (10cm)
the latent phase ( onset to <3cm, 8hs, <16hs)
the active phase (>3cm to 10cm, 4hs, < 8hs)
 The second stage:
---full cervical dilation to the delivery of the fetus, <2hs
 The third stage:
---delivery of the fetus to expulsion of the placenta, <30m’
 The fourth stage:
---observation for at least 1h
Uterine contraction: main
 Rhythm
 Symmetry
 Polarity
 Retraction
The essential factors of labour
A. The power of delivery:
From Uterine contraction
 Rhythmic (contraction-ascending, acme, descending: relaxation)
 Symmetrical,
 Polarity,
 Retraction
B. The passage (the pelvis)

SouNok
C. The passenger (the fetus)
D. The psychic (the mother)
The birth canal;
 Bony pelvis:
 Pelvic planes and line:
 Pelvic inlet
Obstetric conjugate (11cm): from the sacral promontory to
the pubic symphysis
Transverse of pelvic inlet (13cm): left-right diameter
The obliques of pelvic inlet (12.7cm)
 Midpelvis
 Pelvic outlet

What are the seven (7) passive movements of the fetus’ presentation?
Engagement, Descent, Flexion, Internal rotation, Extension,
External rotation and restitution, Expulsion
Labour mechanism of occipital presentation
Engagement → Descent→ Flexion→ Internal rotation→ Extension→
Restitution& External rotation→ Fetus delivery
Engagement
 In the primigravida: occurs late in pregnancy commonly in
the last 2 weeks.
 In the multiparous patient: occurs with the onset of labor.
 Through the biparietal diameter : pelvic inlet
plane→occipito frontal diameter → bi-ischial diameter
Flexion: fetal presentation →to the floor of pelvis→Levator ani muscle
→flexion→occipito frontal diameter →suboccipito-bregmatic diameter
(smallest)

Internal rotation: with the descent of head into the midpelvis, rota¬tion
occurs. The sagittal suture occupies the antero-posterior diameter.
Internal rota¬tion normally begins with the presenting part at the level
of the ischial spines.

SouNok
The levator ani muscles form a V-shaped sling that tends to rotate the
occiput anteriorly.
In cases of occipito-anterior occipital, the head has to rotate 45 degrees,
to pass beneath the pubic arch.

Extension:
Because the vaginal outlet is directed upward and forward, extension
must occur before the head can pass through it.
As the head continues its descent (Uterine and Levator ani muscle
contraction) there is a bulging of perineum .Fur¬ther extension
follows extrusion of the head beyond the introitus.

Gyne final yr q docx


1.Fertilization:it is the process of fusion of the capacitated sperm
with the mature ovum
 It begins with sperm -egg collision and ends with production of
zygote. (a mononucleated single cell .
 It happens 12 hours after ovulation and last about 24hours.

2.Ectopic pregnancy: A fertilized ovum implants in an area other


than the endometrial lining of the uterus.
More than 95% of extrauterine pregnancies occur in the fallopian
tube.
3.Placenta previa: it is defined as the abnormal implantation of
the placenta over the internal cervical os.
When gestation is over 28 weeks, the placenta attaches to the low
segment of uterus,partially or completely covering the internal os
of cervix, the position below the fetal presention.
previa denotes the position of the placenta in relation to the
presenting part.

4.Postpartum hemorrhage:Be defined as a blood loss exceeding 500ml after


delivery of the infant.

5.Abortion:is termination of pregnancy before 20 weeks of gestation and the fetal


weight is less than 500g.

6.Infertility:Inability to conceive after one year of unprotected intercourse

SouNok
7.Ectopic pregnancy:A fertilized ovum implants in an area other than the endometrial lining
of the uterus.

8.Generally contracted pelvic:Each pelvic plane is 2 cm less than normal value or


more,which is called generally contracted pelvic.

9. Menopause:The human ovary gradually becomes unresponsive to gonadotropins with


advancing age,and its function declines,so that sexual cycles and menstruation disappear.

10. Sencondary infertility: infertility that occurs after previous pregnancy regardless of
outcome

11. Normal Menstruation (periodic)

 The origin of menstrual blood: Predominantly arterial.


 Composition: tissue debris、 prostaglandins, fibrinolysin
(unclot).
 Duration of the menstrual cycle: 1to 8 days (3-5days).
 The amount of lost blood: Less than 80ml (30ml).

12.Secondary amenorrhea:is the absence of menses for more than 6


months or for the equivalent of three menstrual cycles in a woman who
previously had menstrual cycles.

13.Endometriosis:A disorder in which abnormal growth of tissue,


histologically resembling the endometrium, are present in locations
other than the uterine lining

14.Placenta abruption: is the premature separation of the


normally implanted placenta from the uterine wall, resulting in
hemorrhage between the uterine wall and the placenta.
after 20 weeks of gestation or during the course of delivery, the
separation of the placenta from its site of implantation before the
baby is delivered resulting in hemorrhage→fetal distress/death.

15.Inevitable abortion:bleeding with dilatation of the cervix


without expulsion of the products of conception. with or without
rupture of the membranes

16.Molar pregnancy(hydatiform mole):( Molar pregnancy is an


abnormal form of pregnancy in which a non-viable fertilized egg implants in the
uterus and will fail to come to term.
A molar pregnancy is a gestational trophoblastic disease which grows into a mass in
the uterus that has swollen chorionic villi. These villi grow in clusters that resemble
grapes.) in net

In ppt..
Hydatidiform mole:
• Molar pregnancy

SouNok
• After pregnancy, syncytiotrophoblastic and cytotrophoblastic
cells proliferate,
• edema of stroma,
• hydropic villi to form mole(bubble)
Complete moles (classic moles) 90%
• It is result of molar degeneration but have no associated fetus
Incomplete moles (partial moles)10%
• It is the result of molar degeneration in association with an
abnormal fetus

17.Amniotic fluid embolism:It is a life-threading complication


that amniotic fluid entering maternal circulation ,result in acute
pulmonary embolism, anaphylactic shock, DIC, renal dysfunction and
sudden death .

18.Abnormal labor curve:


Prolonged latent phase >16hr
(1 cm/2-3hr, 8h, <or equal to 16 h)
Prologed active phase >8hr
Primipara <1.2cm/hr
Multipara <1.5cm/hr
(4h, <or equal to 8hr)
19.Spontaneous Abortion:It is defined as delivery occurring
before the 20th completed week of gestation. It implies delivery of
all or any part of the products of conception, with or without a
fetus weighing less than 500 grams. (The most common complication of
pregnancy)

20.Habitual abortion(recurrent abortion):3 or more consecutive


pregnancy losses each with a fetus weighing <500 g.
21. Complete abortion: the expulsion of all of the products of
conception; bleeding with cervical dilatation and closure.

22.Puberty:is the period when the endocrine and gametogenic


functions of the gonads first develop to the point where reproduction
is possible

23.Inture labour:

SouNok
24.Complete or central placenta previa:The placenta completely
covers the internal os of cervix at the time of termination of
pregnancy is called complete placenta previa.
Onset- Earlier, 28th week
Amount- Severe, shock
Frequency- more

25.MEIGS SYNDROME★:The triad of an ovarian Fibroma, ascites, and


hydrothorax is known as Meigs syndrome.
26.PROLONGED SECOND STAGE:defined as for nulliparous women > 3 hours with
epidural or > 2 hours without; multiparous women > 2 hours with epidural or > 1 hour
without. (15) Otherwise, delivery was designated as within guidelines. (In net)

In ppt…
 Prolonged second stage:
≥2hr in primipara
≥1hr in multipara
1.Postpartum hemorhage causes
 Uterine atony (50%)

 Obstetric lacerations (20%)

 Retained placental tissue (5~10%)

 Coagulation defects

2.Types of sponatneous abortion


• Threatened abortion: bleeding with or without uterine
contractions, no cervical dilatation and no expulsion of the
products of conception.
• Inevitable abortion: bleeding with dilatation of the cervix
without expulsion of the products of conception. with or
without rupture of the membranes
• Complete abortion: the expulsion of all of the products of
conception; bleeding with cervical dilatation and closure.
• Incomplete abortion: the expulsion of some, but not all of
the products of conception. Generally, bleeding is persistent
and is often severe; prolonged cramps are usually present. The
fetus and placenta are usually passed together <10 weeks'
duration. >10 weeks, they may be passed separately with a

SouNok
portion of the products retained in the uterine cavity.
• Missed abortion: the embryo or fetus dies and is retained in
utero(no heart motion)
• Septic abortion :infection of the uterus and sometimes
surrounding structures
• Recurrent abortion: 3 or more consecutive pregnancy losses
each with a fetus weighing <500 g.
• Blighted Ovum: a failed development of the embryo only a
gestational sac, with or without a yolk sac

3.Labor mechanism of occipital presentation/Mechanism of Labor


Engagement → Descent→ Flexion→ Internal rotation→ Extension→ Restitution& External
rotation→Fetus delivery

Engagement:This occurs at various times before the forces of labor begin.

Descent:This occurs as a result of active forces of labor.

Flexion :the fetus neck vertebra further flexed,and the chin approach the chest

Internal Rotation.This occurs as a result of impingement of the presenting part on the bony
and soft tissues of the pelvis.

Extension.:This is the mechanism by which the head normally negotiates the pelvic curve.

External Rotation(Restitution):This is the spontaneous realignment of the head with the


shoulders.

Expulsion:This is anterior and then posterior shoulders, followed by trunk and lower
extremities in rapid succession

4. LIGAMENT OF UTERUS
 Round ligament: Originates from the corni of uterus, Runs through
the inguinal canal to end at the mon pubis. Maintains the uterus in
the anteversion state
 Broad ligament: Made up of only peritoneum. vessels and uteters
pass through its anterior and posterior walls, it gives minimal
support to the uterus
 Cardinal ligament: Fixes the cervix to the pelvis. It is the primary
support of the uterus, helps in anteversion
 uterosacral ligaments: draws the cervix backward and upward, also
maintains anteversion
 vesicocervical ligament (fascia)

5.Implantation favouring conditions


Essential condition of Implantation

SouNok
 the disappearance of zona pellucida
 syncytiotrophoblast is derived from the cytotrophoblast
 The synchronized development of blastocyst and endometrium
function is in coordination.
 The pregnant women can produce enough progesterone.

6.Factors of labour:
The essential factors of labour

 The power of delivery:


From Uterine contraction
o Rhythmic (contraction-ascending, acme, descending:
relaxation)
o Symmetrical,
o Polarity,
o Retraction
 The passage (the pelvis)
 The passenger (the fetus)
 The psychic (the mother)

Powers of uterine contractility and it characterististics


 The power of delivery:
From Uterine contraction
o Rhythmic (contraction-ascending, acme, descending:
relaxation)
o Symmetrical,
o Polarity,
o Retraction

7.How many types of pelvis are there? What are they?


①Gynecoid pelvis ②Android pelvis ③Anthropoid pelvis④ Platypelloid pelvis

8. The most common Complications of Premature separation of the


placenta?
①DIC②Hypovolemic shock③Amnionic fluid embolism④Acute renal failure

9.The attention of using Magnesium sulfate?


①the patient should be checked every 4 hours to be sure that deep tendon reflexes are
present.

②respirations are at least 12/min.

③urine output has been at least 100 mL during the preceding 4 hours.

SouNok
④the antidote for magnesium sulfate overdose is 10 mL of 10% calcium chloride or
calcium gluconate given intravenously. The remedial effect occurs within seconds.

10.What are the characteristics of uterine contractility?


①regularity ②symmetry③polarity ④ retraction

11.The primary function of the placenta is the transport of oxygen and


nutrients to the fetus and the reverse transfer of C02 , urea , and other
catabolites back to the mother.the ways of transfer contains?:
①simple diffusion ②facilitated diffusion ③active transport④Others—pinocytosis

12.The degree of the perineal and vaginal laceration?


①The first degree tear: involves only skin and a minor part of the perineal body

②the second degree tear:involves the perineal body and vagina

③the third degree tear:involves the anal sphincter and anal canal

13.Please describe the complications ART(Assisted Reproductive


Technologies
①Multiple gestations②Pre-eclampsia③Ovarian hyperstimulation syndrome
(OHSS);④Premature birth⑤Low birth weight⑥Long term emotional, social and
psychological impact

14.Ligaments of ovary
Infundibulopevic ligament
Ovarian ligament
Suspensory ligament
Broad ligament

15.Dysfunctionl uterine bleeding classification


 By reason :spontaneous rupture
injured rupture
 By time : rupture during pregnency
rupture during delivery
 By level of rupture :complete rupture
incomplete rupture
 By the position of rupture :
rupture at body of uterus

SouNok
rupture at lower uterine segment

0r pathological classification
1. Endometrial hyperplasia
1. Simple hyperplasia
2. Complex hyperplasia
3. Atypical hyperplasia
2. Proliferative phase endometrium
3. Atrophic endometrium

16.Structure and function of placenta


Structure:
 It is a discoid, deciduate, hemochorial chorioallantoic
placenta.
 fetal surface and matenal surface
 The fetal surface: is covered by the smooth and glistening amnion
with the umbilical cord attached at or near its center. Branches of
the umbilical vessels are visible beneath the amnion as they radiate
from the insertion of the cord
 Maternal surface: showing shaggy look with cotyledons limited by
fissures

Placental function

 Metabolic function: Transfer of nutrients and waste products


between the mother and fetus. In this respect, it attributes to
the following functions: Excretory; Respiratory; Nutritive

Mechanisms involved in the transfer of substances across


the placenta are:
 Simple diffusion: (O2、CO2. H2O,electrolyte)
 facilitated diffusion: glucose
 active transport: amino acids, water-soluble vitamin,
Ca+2,Fe+
 Others--pinocytosis; exocytosis; leakage: macro protein,
immunoglobulin
 Endocrine function: Placenta is an endocrine gland. It
produces both steroid and peptide hormones to maintain

SouNok
pregnancy.
 hCG: Concentrations of hCG rise 6 days after fertilization
exponentially until 9-10 weeks' gestation, with a doubling
time of 1.3-2 days. hCG will disappear 2 weeks after delivery
( B-hCG can detect pregnancy 10 days after fertilization). ,
 HPL,
 Estrogen,
 Progesterone, B1 glycoprotein,

 Barrier function.

 Immunological function

17.Describe the signs of labour


(i) Painful uterine contractions at regular intervals
(ii) frequency of contractions increase gradually
(iii) intensity and duration of contractions increase progressively
rhythm and gradually increase the uterine contraction, and lasted
for 30 seconds or more, intermittent 5 to 6 minutes;
(iv) progressive effacement and dilatation of the cervix
(v) descent of the presenting part
(vi) did not relieved by sedatives.

18.Three periods of pregnancy (doubt)


during the 32nd-34th gestational weeks, the blood volume increases to
maximum (30-40 %↑)
• the burden of the heart is heaviest during delivery, the first stage
is caused by uterine contraction (blood enters the systemic
circulation from the uterus)
• second stage when the breath is held: peripheral resistance ↑
• Third stage after delivery of the placenta, the blood volume
increases suddenly.

Cardiovascular changes in normal pregnancy

 gestational period
 During delivery
 Puerperium
Returned blood volume increased in 3 days in postpartum and
return to normal after 2~6 weeks.

21.Complication of placental abruption

SouNok
 fetal death
 DIC
 postpartum hemorrhage
 Acute renal failure
 Amniontic fluid embolism

22.Rupture of uterus etiology


1 Obstructive dystocia
2 Injured rupture of uterus
3 scar uterus
4 previous uterine curettage or perforation
5 The misuse of hysterotonics
the counterindication of Oxytocin(缩宫素):
cephalopelvic disproportion,
fetal distress,
scar uterus,
mal match uterine contraction,
Excess extend of uterus body

23.Factors causing infertility


1.ovulatory disorders (32%)
2.fallopian tube abnormalities - pelvic adhesions (34%)
endometriosis(15%)
3.Other factors uterine and cervical factors
luteal phase defect
genetic disorders

24.Composition of placenta
• Amnion: fetal surface
• Chorion frodosum: attached to the basal deciduas
• Basal decidua

25.Complication of ectopic pregnancy


tubal abortion
tubal rupture

SouNok
tubal mole
Secondary Abdominal Pregnancy
Ovary pregnancy
Abdominal pregnancy
Cervical pregnancy
Broad ligament pregnancy
(may b)this too…
 amenorrhea : 6-8w,may be absent even

 abdominal pain: Main symptom


 abnormal vaginal bleeding:slight,contineous
Syncope and shock
Other symptoms: nausea, vomiting

26.Main reason can lead to ectopic pregnancy?


“Block” – the passage
“Delay” – the march
chronic salpingitis: most common
congenital tubal abnormalities
endometriosis
IVF-ET
IUD
tumor

27.Surgical indications of pelvic inflammatory disease


 Indications

-- failure of medical therapy(48-72h)

-- ruptured TOA/ pelvic abscess

-- persistent pelvic abscess

 Surgery

-- percutaneous drainage/colpotomy (posterior fornix)

-- laparotomy:

salpingectomy/ salpingo-oophorectomy

SouNok
28. Natural defence function of female genital tract
• Vulvar--bilateral labium majus closed

• Vaginal is a potential cavity with posterior and anterior walls sticking tightly,
Estrogen thickens the vaginal epithelium and results in large quantities of glycogen
which results in the production of lactic acid. This acid environment (pH of 3.5-4.0)
promotes the growth of normal vaginal flora, chiefly lactobacillus--------self cleaning

• Cervical canals closed and mucus plug

• Endometrium: menstruation (reproductive age female)

• Fallopian tube---peristalsis

• Immune system

29.Complications of OF ovarian tumour


A. Torsion of the pedicle

B. Rupture of tumor

C. Infection

D. Malignant change

(Write some epln fr each)

Or,
A. Torsion of the pedicle:

1. Common gynecologic emergency.

A. medium-sized tumors, pedicle is long, the center of gravity is instable;

B. During pregnancy and puerperium, due to uterine position change, or patients change
position suddenly;

C. When acute torsion happens,the venous return from the cyst is occluded→
hemorrhage and hemotoma in tumors→the tumors will enlarge quickly.

if the arterial return from the cyst is occluded → necrosis, rupture and secondary
infection of the tumors.

2. Symptoms:

Acute severe pain with nausea and vomiting

3. Signs:

Tumor enlarged and tense ,tender, severe pain over pedicle.

※4. Treatment: Once confirmed diagnosis, surgery immediately

--unilateral salpingo-oophorectomy(benign)

SouNok
caution:Pedicle should not be restored in order to prevent embolic thrombosis.

pathology during operation

B. Rupture of tumor:

spontaneous rupture: malignant tumor

traumatic rupture:

abdominal pain

intra-abdominal hemorrhage

surgery if suspected

pathology during operation

C. Infection:

Rare,secondary to torsion and rupture

symptoms like those of the ordinary type of acute pelvic inflammatory disease.

D. Malignant change:

Bilateral, grow rapidly→surgery

30.Metastatic pathway of endometrial cancer


Direct extension-Invasion of the myomatrim is the initial form of spread.It is
believed that the tumor remains confined to the body of the uterus for a relatively
long time, but eventually it invades the myometrium and cervix. It may then spread
to the parametrial, the pelvic wall and aortic nodes, the serosa(浆膜) of the
uterus, the ovaries, and ultimately the peritoneal surfaces.

Lymphatic spread-It is the main way of spread

Pelvic lymph node,

Para-aortic lymph node

Hematogenous spread-Hematogenous metastases to the lungs ,bone or liver are


uncommon with primary tumors limited to the uterus but do occur with recurrent or
disseminated disease.

31.Symptoms of cervical cancer


Clinical findings

• Usually no symptoms or signs

• Early detection is extremely important

early stage: cervix is smooth or columnar ectopy.

later: Post-coital bleeding

. the diagnosis is most often based on biopsy findings following an abnormal routine

SouNok
cervical cytology smear

Because high-grade dysplasia probably is a transitional phase in the pathogenesis of


many cervical cancers, early detection is extremely important. All women who have
been sexually active or reached age 18 should have a pelvic and cytologic examination at
least once a year. The screening interval may be extended in selected low-risk patients.
However, annual screening should be continued in women with any risk factors for CIN.

32.Surgical indications of uterine myoma


① Abnormal uterine bleeding, causing anemia.

② Severe pelvic pain.

③ Pressure-related symptoms--Urinary frequency, retention, hydronephrosis or


constipation.

④ The only reason of recurrent miscarriage or infertility.

⑤ be suspicious of Sarcomatous degeneration --Growth after menopause,Rapid


increase in size

The choice of surgery

1. Myomectomy:wish to preserve their fertility, myoma resection

2. Hysterectomy:It is the definitive treatment

by laparotomy or laparoscopically

33.Indications of prophylactic chemotherapy in molar pregnancy


Prophylactic chemotherapy

high risk or poor follow-up

high risk:1、Beta-hCG>100,000U/L

2、Larger than expected for the gestational date

3、Theca lutein cysts>6cm

4、>40Y or recur

Suspected metastasis:

 MTX、Dactinomycin, single drug one course


 Surveillance toxicity

34.Classification of uterine myoma and myometrium


 According to the relationship between myoma and uterine myometrium:

---Submucous myoma(10%~15%) : lie just beneath the endometrium and grow toward
the uterine cavity. pedunculated.

---Intramural myoma(60%~70%): lie within the uterine wall and are completely

SouNok
surrounded by normal myometrim.this’s the most common type

---Subserous myoma(20%):lie just beneath the serosal surface of the uterus .

Pedunculated

35.Follow up and treatment of cervical cancer


• CINI:

60% can remove by itself---follow up .

review 6-12 monthes.

≥2 years---treatment : Electrocoagulation, cryotherapy or laser ablation

Electrocoagulation uses temperature over 700°C and destroys the tissue up to 8–10 mm
deep. Since the procedure is painful, it is done under general anaesthesia. Recurrence,
bleeding, sepsis and cervical stenosis are its complications. Squamocolumnar junction gets
indrawn within the cervical canal.

• CIN Ⅱ:

20% progress to CIN Ⅲ, 5% progress to cervical cancer

LEEP(loop electrosurgical excision procedure) or cold knife conization

• CIN Ⅲ:

cold knife conization or hysterectomy

36.Types of degeneration
Defn: lose the original typical structure.

 The following types:※

1. Hyaline degeneration

2. Cystic degeneration

3. Red degeneration

4. Sarcomatous change(Malignant Transformation)

5. Degeneration with calcification

37.Categories of endometrial hyperplasia


 Simple hyperplasia

with nuclear atypia

without ~

 Complex hyperplasia

with nuclear atypia

SouNok
without ~

OBSTETRICS AND GYNECOLOGY MID


TERM 2018
ANATOMY OF THE FEMALE REPRODUCTIVE SYSTEM

1. Female external genitalia (vulva, pudendum)


 Mons pubis: subcutaneous adipose connective tissue

 Labia majora: joins medially to form the posterior commissure in front of the anus.
Outer skin is pigmented, with an adipose tissue richly supplied with venous plexus
that can form a hematoma if injured (homologous to the male scrotum). The round
ligament terminates at the upper border.

 Labia minora: no fat, hair follicles or sweat glands. Divide to enclose the clitoris and
unite with each other in front and behind the clitoris to form the prepuce and
frenulum respectively.

 Hymen
 Clitoris: homologous to the penis, attached to the surface of the pubic symphisis by
a suspensory ligament

 Vestibule: triangular space bounded by the clitoris anteriorly, posteriourly by the


fourchette and on either side by the labia minora. Consists of four openings;
urethral, vaginal, bartholins gland and skene’s gland.

 Urethra
• Skene’s glands: homologous to the prostate

• Bartholin’s glands

• Vestibular bulbs

2. The blood supply of the external genitalia


comes from the branches of the (a) internal pudendal artery (b) branches of femoral artery.
The venous drainage is to the internal pudendal vein, long saphenous vein and the
vaginal/vesical venous plexus

3. Nerve supply to the external genitalia


cutaneous branch from the ilioinguinal nerve, genital branch of the genitor-femoral nerve
(L1-L2)

SouNok
4. Internal genital organs
 Vagina (diameter 2.5cm): posterior wall 9cm long and anterior wall 7cm. the
vaginal PH from puberty to menopause is acidic because of the presence of
Doderlain’s Bacilli which produces lactic acid from the glycogen in exfoliated cells.
Arterial supply of the vagina is from the cervicovaginal artery from uterine artery;
vaginal artery from the anterior division of the internal iliac artery; middle rectal
artery and internal pudendal artery. All form the azygos arteries by anastomosing.
Venous drainage is to the internal pudendal veins.

 Uterus: normal position is anteversion- anteflexion, usually inclines to the right


(dextrorotation) →pelvis to the left (levorotation). 8cm long, 5cm wide, 50-80g in
weight.Consists of the body (fundus, cornuaattachment of the tubes, round
ligaments and ovarian ligaments), isthmus (constricted part-limited above and below
by the anatomical and histological Os respectively, Site for CS), and cervix
(insensitive to touch, heat). The cavity is triangular shaped, no cavity in the fundal
part making the length of the cavity 6.5-7cm.

 The perimetrium is the serous coat that invests the entire uterus except on the
lateral surface

 The myometrium consists of 3 muscle layers: outer longitudinal, middle interlacing


and inner circular

 Endometrium: mucous lining (lamina propria and surface epithelium), no


submucous layer. The endometrial secretion is alkaline.

 Uterus nerve supply (T5-T6- motor, T10-L1-sensory)

 Fallopian Tube: 8-14cm in length consists of the intramural/interstitial (Shortest


and narrowest 1cm), isthmus (3-4cm), ampulla (widest, tortuous 5-8cm), and
fimbrae.

 Ovary: 3cm in length, 2cm in breadth and 1cm in thickness. Attached to the lateral
pelvic wall by the infundibulopelvic ligament

Functions: germ cell maturation, storage and egg release, steroidogegesis (hormones)

5. types of pelvis:
• gynecoid,

• platypelloid,

• android,

• mixed type

6. The pelvic floor 3 layered muscle


 Outer layer (4 muscles)
• Bulbo carvenosus muscle

SouNok
• schiocavernosus

• uperficial transverse perineal muscle 


External sphincter ani

 Middle layer
 Urogenital diaphragm

 inner layer
 levator ani muscles: pubococcygeus,

iliococcygeus, ischiococcygeus The rectouterine pouch is the lowest

part of the female pelvis

7. The Main ligaments of the uterus


 Round ligament: Originates from the corni of uterus, Runs through the inguinal
canal to end at the mon pubis. Maintains the uterus in the anteversion state

 Broad ligament: Made up of only peritoneum. vessels and uteters pass through its
anterior and posterior walls, it gives minimal support to the uterus

 Cardinal ligament: Fixes the cervix to the pelvis. It is the primary support of the
uterus, helps in anteversion

 uterosacral ligaments: draws the cervix backward and upward

8. Joints of the pelvis (false & true pelvis)


• Sacroiliac joint

• Sacrococygeal joint

• Pubis symphysis

9. Pelvic ligaments
 Sacrotuberous ligaments

 Sacrospinous ligaments (from lateral border of sacrum to ischial spine)

PHYSIOLOGY OF THE FEMALE REPRODUCTIVE


SYSTEM
1. The polar bodies are located in the perivitelline space

2. Sperm capacitation: the physiochemical change in the sperm by which it


becomes hypermotile and is able to bind and fertilize a secondary oocyte.

3. Acrosomal reaction: activation of acrosomal enzymes which releases enzymes

SouNok
that help the sperm to digest the zona pellucida and to enter the oocyte

4. zona reaction: prevents polyspermy

5. Ovulation: a process in which a secondary oocyte is released from the ovary


following rupture of a mature graafian follicle and becomes available for conception

6. morula: 16-64 cell stage that enters the uterine cavity

7. Implantation: occurs on the 6th day through four stages; apposition, adhesion,
penetration and invasion

8. decidual reaction: the increased structural and secretory activity of the


endometrium that is brought about in response to progesterone following implantation.
Three layers of the decidua: superficial spongy layer; spongy layer; thin basal layer

9. Endometrium: the superficial 2/3 is the stratum functionale shed during


menstruation and supplied by the spiral artery. Deep 3rd stratum basale, supplied by
straight basilar arteries

10. 3 phases of uterine cycle


 Proliferative phase(preovulatory/ follicular):under the influence of estrogen

 Ovulatory phase

 Secretory phase/luteal phase: the length is constant, at 14days

11. Cyclic change in the breast: estrogen leads to proliferation of the


ducts while progesterone leads to lobular and alveoli maturation 12.
Indicators of ovulation: rise in basal body temperature and LH rise
13. Different periods of the female life
VI. Neonatal period : £ 4 weeks

VII. childhood: 4 weeks to age of 12

VIII. Adolescence or Puberty

IX. sexual maturity: maintains for 30 years

X. peri-menopausal period: begin 40, maintains for 10-20 years

Pre-menopause, menopause (last time of menorrhae), post-menopause

Vi. senility

14. Puberty: Puberty is the period when the endocrine and gametogenic functions of
the gonads first develop to the point where reproduction is possible

SouNok
15. Puberty Events
e. Thelarche: the development of breasts.(E,P)

f. Pubarche: the development of axillary and pubic hair.(A)


g. Menarche: the first menstrual period.

h. Adrenarche: an increase in the secretion of adrenal androgens.

16. Delayed or absent puberty:


 Definition: considered when menarche has failed to occur by the age of 17.
 Reasons may be due to abnormalities in:
 Hypothalamus

 pituitary (dwarfing + endocrine abnormalities)

 Idiopathic: In some individuals, puberty is delayed even though the gonads are
present and other endocrine functions are normal.

17. MENOPAUSE

 Conception:
Gradual unresponsiveness of the ovaries to gonadotropins with advancing age,
decreased negative feedback of the HPO which leads to its decline in function, so that sexual
cycles and menstruation disappear.

 Age: The menses usually become irregular and cease between the age of 45 and
55.
 Symptoms:
• The uterus and vagina gradually become atrophic.

• Sensations of warmth spreading from the trunk to the face (hot flash), night sweats,
and various psychic symptoms.

18. Normal Menstruation (periodic)


 The origin of menstrual blood: Predominantly arterial.

 Composition: tissue debris、 prostaglandins, fibrinolysin (unclot).

 Duration of the menstrual cycle: 1to 8 days (3-5days).

 The amount of lost blood: Less than 80ml (30ml). Menstrual cycle

E.g.: LMP 25 Aug. cycle: 28days

25 + 28 = 53→53- 31days = 22 sep. (next MP)

Ovulation
E.g. LMP 15 Feb. Cycle: 35days, present date 21st Feb Ovulation; 21 + 15=36 – 29= 7th
march

SouNok
19. The follicle cells secrete estrogen while the luteal cells secrete
estrogen and progesterone.
Follicles: 7 million (fetal), 2million (at birth), 300-500,000 (puberty). GnRH release is episodic

20. Cyclic changes


 uterine cycle

 uterine cervix cycle

 vaginal cycle

 the cycle of breasts  indicators of ovulation

21. anovulatory cycle


 the first 12-18months after menarche and onset of menopause

 no corpus luteum is formed→no progesterone

 estrogen continues to cause endometrial growth

 the proliferated endometrium becomes thick enough to slough→bleeding occurs

22. Hypothalamus pituitary ovarian pathway


 hypothalamus secretes GnRH →stimulates FSH and LH release from the pituitary

 before ovulation, there is a surge in estrogen but progesterone remains very low

 after ovulation, the corpus luteum is formed and it secretes progesterone and
estrogen both of which have a surge

 in the entire cycle, estrogen has two surges while progesterone has one surge

 no pregnancy right before ovulation because of the absence of progesterone

 both FSH and LH have a surge and LH surge triggers ovulation and very high FSH
surge occurs before ovulation→ stimulates the follicles

23. Effects of estrogen

PREGNANCY PHYSIOLOGY

2. Pregnancy: the maternal condition of having a developing fetus in the body


beginning with fertilization and ends with expulsion of the fetus

3. Fertilization: it is the process of fusion of the capacitated sperm with the mature
ovum
 It begins with sperm -egg collision and ends with production of zygote. (a
mononucleated single cell .

 It happens 12 hours after ovulation and last about 24hours.

SouNok
3. Stages of fertilization
Capacitation: in female reproductive tract, the sperms are enabled to bind to the zona

pellucida receptors. 4. Acrosome reaction: rupturing of the acrosome releases


the enzymes that dissolve the zona pellucida enabling the sperm to get in

6. zona reaction: the zona pellucida rebuilds, preventing other sperms from going
in (destruction of the sperm receptors in the zona pellucida).

7. Implantation occurs through three stages:


• Apposition

• adhesion,

• Penetration and Invasion.

7. Essential condition of Implantation


 the disappearance of zona pellucida

 syncytiotrophoblast is derived from the cytotrophoblast

 The synchronized development of blastocyst and endometrium function is in


coordination.

 The pregnant women can produce enough progesterone.

8. Decidual reaction: The increased structural and secretory activity of the


endometrium that is brought about in response to progesterone following implantation is
known as decidual reaction

• Basal decidua: the portion of the decidua in contact with the base of the blastocyst,
where the zygote is implanted. It is the maternal part of placenta.

• capsular decidua: the thin superficial compact layer covering the blastocyst

• true decidua: the rest of the decidua lining the uterine cavity outside the site of
implantation

9.
• 16 weeks; the sex is discernible as male or female.

• 20 weeks; Heart tones may often be detected by stethoscope. Movements have


been perceived by the mother. The uterine fundus is near the level of the umbilicus.

• 40 weeks: The term fetus averages 50 cm in length and 3000 g in weight. The head
has a maximum transverse (biparietal) diameter of 9.5 cm. The average fetus
therefore, requires cervical dilatation of almost 10 cm before it can descend into the

SouNok
vagina.

10. The placenta: fetal surface and maternal surface


 The fetal surface: is covered by the smooth and glistening amnion with the umbilical
cord attached at or near its center. Branches of the umbilical vessels are visible
beneath the amnion as they radiate from the insertion of the cord

 Maternal surface: showing shaggy look with cotyledons limited by fissures

 Compositions

• Amnion: fetal surface

• Chorion frodosum: attached to the basal deciduas

• Basal decidua

11. placenta-blood circulation


Fetal and maternal blood in their respective vascular systems, without communication.

• Maternal: spiral A→intervilli space→ spiral V→maternal material exchange

• Fetus: umbilical A→villi capillaries→umbilical V→fetus

12. Placental function


 Metabolic function: Transfer of nutrients and waste products between the mother
and fetus. In this respect, it attributes to the following functions: Excretory;
Respiratory; Nutritive
Mechanisms involved in the transfer of substances across the placenta are:

 Simple diffusion: (O2、CO2. H2O,electrolyte)

 facilitated diffusion: glucose

 active transport: amino acids, water-soluble vitamin, Ca+2,Fe+

 Others--pinocytosis; exocytosis; leakage: macro protein, immunoglobulin

 Endocrine function: Placenta is an endocrine gland. It produces both steroid and


peptide hormones to maintain pregnancy.

 hCG: Concentrations of hCG rise 6 days after fertilization exponentially until 9-10
weeks' gestation, with a doubling time of 1.3-2 days. hCG will disappear 2 weeks
after delivery ( B-hCG can detect pregnancy 10 days after fertilization). ,

 HPL,

SouNok
 Estrogen,

 Progesterone, B1 glycoprotein,  Barrier function.

 Immunological function

14. Fetal membrane (surrounds the fetus): It consists of two layers: outer chorion
and the inner amnion.

Functions:
 Contribute to the formation of liquor amnii;

 Intact membranes prevent ascending uterine infection;

 Facilitate dilatation of the cervix during labor;

 Has got enzymatic activities for steroid hormonal metabolism;

 Rich source of glycerophospholipids containing arachidonic acid —precursor of


prostaglandin E2 and F2α

15. at birth, the mature cord is about 30-100 cm in length and 8-20 mm in
diameter

15. The amniotic fluid


 Sources:
• early trimester: the ultra-filtrate of maternal plasma

• After mid trimester :fetal urine is the main source

• fetal lungs, amnion, fetal skin, Wharton's jelly


 Absorption: fetal membrane (50%); digestive canal, umbilical cord, fetus skin
• The amniotic fluid is continuously
exchanged at a rapid rate  Capacity:
800ml at term, post-term pregnancy<
300ml.
Status of amniotic fluid: pH: 7.20; Density: 1.007-1.025.

 Contents: water(98-99%), inorganic substance,


organic substance  functions:

I. Protects the fetus: It cushions the fetus against severe injury; provides a medium in
which the fetus can move easily ,

II. may be a source of fetal nutrients;

III. prevents infection

SouNok
IV. A source for diagnostic analysis of fetal tissues and fluids.

16. Maternal changes during pregnancy


 Genital Tract (uterus: capacity: 5-10ml→5000ml, weight: 50-70g→1100g):
hypertrophy and hyperplasia.
uterine enlargement is not symmetrical, more at the fundus

 Uterine contractions: (braxton hicks)

 The isthmus becomes softer, longer (from 1cm→7-10cm)

 Corpus luteum becomes cystic

 Breast: enlarges, colostrums and Montgomery formed

 Cardiovascular System: Cardiac capacity increases by 70 - 80 mL(maximum at 35-


37wkss), CO ↑ approximately 40%

 Hematologic system: Plasma volume:↑1000ml; Blood Cells:450ml


(hemodilution.)→anemia, increased clotting factors,

 Pulmonary System: Total lung capacity is reduced (4%-5%) due to the elevation of
the diaphragm; maternal hyperventilation.

 Gastrointestinal Tract: Gastric emptying time is prolonged (nausea, vomiting),


constipation, insulin resistance, slowed gallbladder emptying.

 Urinary system: GFR, RPF↑. More pressure to the right side →(hydronephrosis)

17. Supine hypotensive syndrome: during late pregnancy, the gravid uterus
produces a compression effect on the inferior vena cava when the patient is in supine
position. This results in hypotension, tachycardia and syncope. The normal blood pressure is
quickly restored by turning the patient to lateral position

PLACENTA
PREVIA
1. Placenta previa: it is defined as the abnormal implantation of the placenta over
the internal cervical os.
When gestation is over 28 weeks, the placenta attaches to the low segment of
uterus,partially or completely covering the internal os of cervix, the position below the
fetal presention. previa denotes the position of the placenta in relation to the presenting
part.

SouNok
2. Etiology of placenta previa
 Endometrial factors:

• Scarred or poorly vascularized endometrium in the uterine corpus.

• Endometritis: Puerperal infection、prolificacy、curettage、cesarean section et al®


endometritis, the trauma of endometrium® to take more nutrition ,placental area
enlarged.

 Placental factors:

• Large placenta( multiple pregnancy)

• Abnormalities of placenta (Bilobed placenta, Succenturiate placenta, Horseshoe


placenta, Lingual –shaped placenta, Long and round placenta)

 Zygote factors: the retardation of zygote growth.

High risk factor:


• repeated abortion and curretage,

• the history of cesarean section,

• prolificacy,

• multiple pregnancy

3. Classification
I. Complete or central placenta previa

II. Partial placenta previa

III. Marginal placenta previa

IV. Low lying placenta: Placenta is in lower segment, but the lower edge does not reach
internal os

4. Clinical manifestations
 Symptoms: painless hemorrhage

 Character: sudden onset, painless, apparently causeless and recurrent

 Time: late pregnancy (after the 28th week) and delivery Unrelated to activity, often
occurs during sleep, in a pool of blood.

 Why? The low segment of the uterus prolonged, disappearance of the cervical
canal as well as cervical dilatation, the placenta will not extend, leading to the
rupture of blood sinus.  Signs:

I.

An

SouNok
emi
a
and
sho
ck
II.

Ab
do
min
al
exa
m:

III. The size of uterus is proportionate to the week of


gestation

IV. the uterus is soft, relaxed, elastic, no tenderness V.


fetal mal-presentation and fetal distress

VI. High floating of presenting part

VII. Hear the placental souffle above the pubic symphysis

5. Diagnosis
v. Clinical symptoms and signs

vi. Sonography: the most important method-simple, precise, safe (≥28weeks). <28W,
the uterine size is small and placenta occupies most volume (observation should be
done)

vii. MRI

viii. Diagnoses in parturition

• The shape of placenta

• The distance from edge of placenta to the rupture of the fetal membranes
is less than 7cm • Clot in the maternal side of placenta

6. Differential diagnosis
IV. Placental abruption

V. Rupture of the marginal sinus

VI. Rupture of the vasa previa in cord velamentous insertion IV. Bleeding of cervix and
vagina

7. Complications
 Maternal:

• Postpartum hemorrhage, shock, death

SouNok
• Puerperal infection

• Placenta increta: myometrium penetrated by trophoblast of placenta Thus, the


placenta grows completely through the endometrium.

 Fetus:

• Preterm delivery

• Perinatal mortality rate is increased

8. Treatment
I. Expectant therapy (should not exceed the 36th week)

 Why (preterm, prematurity)

 Indications

• <34 week

• Fetal weight <2000g

• Bleeding not severe

II. Termination of pregnancy

• Severe hemorrhage

• >36weeks GA

• Mature fetal lungs

• Fetal death/distress (34-36wks)

PLACENTAL
ABRUPTION
9. PLACENTAL ABRUPTION Definition: after 20 weeks of gestation or during the
course of delivery, the separation of the placenta from its site of implantation before the
baby is delivered resulting in hemorrhage→fetal distress/death.
Placental abruption is the premature separation of the normally implanted placenta from
the uterine wall, resulting in hemorrhage between the uterine wall and the placenta.

10. Etiology
 Local vascular injury (Hypertensive Disorders Complicating Pregnancy, chronic
hypertension, chronic renal disease)

 Sudden decrease in the pressure of uterus

 Trauma, short cord syndrome

SouNok
11. High risk factors for placenta previa
i. Increased age and multiparity

ii. uterine distention (multiple pregnancy)

iii.vascular deficiency or deterioration ( diabetes mellitus) iv. uterine anomalies or


tumors ( leiomyoma)

v. cigarette smoking, alcohol consumption

vi. Blood clotting disorders (thrombophilias)

12. Types
iv. the revealed abruption ( the external bleeding )

v. The concealed abruption ( the internal bleeding )

vi. mixed bleeding

13. Clinical findings


 Abdominal pain with/without vaginal bleeding

 Degrees

• Mostly occur during delivery. <1/3 abruption

• In the third trimester of pregnancy. ½ abruption(uterus >GA due to


hematoma)

 Abruption >1/2

 Internal hemorrhage

 Persistent abdominal pain

 Shock

 Mostly caused by accident or pre-eclampsia

14. Differential diagnosis


pl

nt

pr

SouNok
e

vi

a

Pr

e-

ru

pt

ur

of

ut

er

15. Complications
i. fetal death, DIC

ii. postpartum hemorrhage

iii.Acute renal failure iv. Amniontic fluid embolism

16. Treatment
Principle: timely diagnosis, once the diagnosis of placental abruption is made, Termination of
pregnancy timely.

★ Treatment of hypovolemic shock :intensive transfusion with blood

★ Termination of pregnancy timely : Vaginal delivery, CS

★ treatment of complications

• Postpartum hemorrhage

• Coagulation dysfunction 、DIC

• Acute renal failure

SouNok
SPONTANEOUS ABORTION
1. Definition: It is defined as delivery occurring before the 20th completed week of
gestation. It implies delivery of all or any part of the products of conception, with or without
a fetus weighing less than 500 grams. (The most common complication of pregnancy)

2. Etiology
 Morphologic and Genetic Abnormalities (Aneuploidy> 50%)

 Maternal Factor

 Systemic Disease

 Uterine Defect

 Immunologic Disorder  Malnutrition, toxins, trauma

3. Pathology of Abortion
iii.Hemorrhage( into the decidua basalis).

iv.Necrosis and inflammation appear in the area of implantation. iii.


Detachment of product of conception (partially or entirely).

vi. Uterine con¬tractions and dilatation of the cervix

vii. Expulsion off all or some of the products of conception

4. Types of Abortion
ix. Threatened abortion: bleeding with or without uterine contractions, no cervical
dilatation and no expulsion of the products of conception.

x. Inevitable abortion: bleeding with dilatation of the cervix without expulsion of the
products of conception. with or without rupture of the membranes

xi. Complete abortion: the expulsion of all of the products of conception; bleeding with
cervical dilatation and closure.

xii. Incomplete abortion: the expulsion of some, but not all of the products of
conception. Generally, bleeding is persistent and is often severe; prolonged cramps
are usually present. The fetus and placenta are usually passed together <10 weeks'
duration. >10 weeks, they may be passed separately with a portion of the products
retained in the uterine cavity.

xiii. Missed abortion: the embryo or fetus dies and is retained in utero(no heart motion)

xiv. Septic abortion :infection of the uterus and sometimes surrounding structures

xv. Recurrent abortion: 3 or more consecutive pregnancy losses each with a fetus
weighing <500 g. viii. Blighted Ovum: a failed development of the embryo only a
gestational sac, with or without a yolk sac

5. Laboratory test
• Complete Blood Count: Anemia, WBC count, ESR.

SouNok
• Pregnancy Tests: Falling or abnormally low plasma levels of β-
hCG  Cervical cultures: To determine pathogens in case of
infection.

6. Complication
o Severe or persistent hemorrhage

o Sepsis, Infection, o Intrauterine


synechia, o Infertility o Perforation 、injury
to the bowel and bladder、hemorrhage,
infection, and fistula formation ( D and C)

7. Treatment
d. Threatened abortion: Bed rest and pelvic rest; Prognosis is good when
bleeding and/or cramping resolve.
e. Incomplete abortion: evacuation of the uterus by suction D and C should be
promptly performed; cross-match for possible blood transfusion and deter-mination
of Rh status should be obtained. The prog¬nosis for the mother is excellent if the
retained tissue is promptly and completely evacuated.
f. Complete abortion: The patient should be observed for further bleeding; the
products of conception should be examined. The prognosis for the mother is
excellent.

• If abortion occurs after the first trimester: hospitalization, Oxytocics, Ergot, D and
should be administered
d. Septic abortion: Hospitalization, Intravenous antibiotic therapy, D and C,
hysterectomy

ECTOPIC
PREGNANCY
1. Ectopic pregnancy: A fertilized ovum implants in an area other than the
endometrial lining of the uterus. More than 95% of extrauterine pregnancies occur in the
fallopian tube.

2. Classification
III. Tubal (> 95%): Includes; ampullary (55%), isthmic (25%), fimbrial (17%), and
interstitial (2%).

IV. Others ( < 5% ): Includes cervical , ovarian , and abdominal ( most abdominal
pregnancies are secondary preg¬nancies, from tubal abortion or rupture and
subse¬quent implantation in the bowel, omentum, or mes¬entery

SouNok
3. Etiology
• Tubal Factors: salpingitis

• Ovarian Factors

• Other Factor: Intrauterine device ( IUD )

• Bleeding is of uterine origin

4. Time of rupture
d. Isthmic pregnancies: the earliest, at 6 to 8 weeks due to the small diameter

e. Ampullary pregnancies: later, at 8-12 weeks.

f. Interstitial pregnancies: the last, at 12-16 weeks as the myometrium allows more
room to grow than the tubal wall.

Interstitial rupture is quite dangerous, as its proximity to uterine and ovarian vessels can
result in massive hemorrhage.

5. Clinical findings
 Bleeding 75% (spotting decidual sloughing)

 Abdominal Pain 100% (unilateral or bilateral / localized or generalized /


subdiaphragmatic or shoulder pain (intra-abdominal bleeding)

 Amenorrhea 50%

 Syncope (30-50%), Dizziness, lightheadedness

 Decidual Cast

Signs
 Tenderness

 Adnexal Mass

 Uterine Changes

6. Culdocentesis: the transvaginal passage of a needle into the posterior cul-de-sac in


order to determine whether free blood is present in the abdomen.

8. Treatment:
E. Expectant Management: when β-hCG titer is low ( < 200 mlU/mL ) or decreasing, and
the risk of rupture is low

F. Surgical Treatment

G. Emergency Treatment: Immediate surgery is indicated when the diagnosis of ectopic

SouNok
pregnancy with hemorrhage is made.

H. Medical Management: small, unruptured ectopic pregnancies in asymptomatic


woman.

HYPERTENSIVE STATE IN PREGNANCY


1. Hypertensive states in pregnancy
 Gestational hy¬pertension (pregnancy-induced hypertension)

 preeclampsia (mild and severe)

 eclampsia

 chronic hypertension ( either essential, or secondary to renal disease, endocrine


disease, or other causes )

 chronic hypertension with superimposed preeclampsia

2. Hypothesis
An immunologic dis¬turbance causes abnormal placental implantation →decreased
placental perfusion→ stimulates the production of substances in the blood →activate or
injure endothelial cells→The vascular endothelium provides a single target for these blood-
borne products, →the mul¬tiple organ system involvement in preeclampsia

3. Pathophysiology
i. Central Nervous System: headache, convulsion, coma

j. Eyes: retinal edema, detachment, blindness

k. Pulmonary System

l. Cardiovascular System: HF/ pulmonary edema

m. Kidney: injury and swelling of the endothelial cells; proteinuria; oligura

n. Liver: HELLP SYNDROME

o. Blood : DIC

p. Placenta-fetus

3. Gestational hypertension
c. Hypertension: Occurs 20weeks after gestation and recover 12 weeks postpartum.
SP≥140mmHg and
(or)DP≥90mmHg

d. proteinuria(-) c.Diagnosed only after delivery

4. Preeclampsia
 Mild-preeclampsia

SouNok
 Hypertension : ≥20 gestational weeks; SP≥140mmHg and (or)DP≥90mmHg

 Proteinuria: urine protein ≥0.3g/24h, urine protein (+)

5. Severe preeclampsia:
k. BP:SP ≥160mmHg and (or)DP≥110mmHg

l. severe headache or visual changes

m. Heart failure or pulmonary edema.

n. epigastric pain; (RUQ pain)

o. elevation of transaminases : ALT、 AST ↑

p. proteinuria ≥ 2 g/ 24h or urine protein ≥(+++)

q. abdominal dropsy or pleural effusion

r. acute renal failure with rising creatinine

s. oliguria : < 400 ml/24 h or <17ml/h

t. Thrombocytopenia (platelet count < 100 ×109/L) 、intravascular hemolysis,


anemia、jaundice、LDH↑. k. before 34 gestational weeks.

6. Eclampsia
C. preeclampsia + seizures

D. Occurrence: prenatal 、intrapartum、 postpartum

 the convulsive movements : suddenly,1-1.5min, Without breathing


• Facial twitchings

• generalized muscular contraction

• jaws close violently

• all muscles alternately contract and relax

• lies motionless

7. Chronic hypertension and pregnancy


 Chronic hypertension: SP≥140mmHg and(or)DP≥90mmHg before pregnancy or
before 20 weeks' gestation persists for more than 12 weeks postpartum.

 proteinuria(-)

8. Chronic hypertension superimposed on preeclampsia


iv. chronic hypertension + proteinuria

v. SP≥140mmHg and(or)DP≥90mmHg, before pregnancy or before 20 weeks'


gestation, persists for more than 12 weeks postpartum.

SouNok
vi. chronic hypertension + higher BP iv. thrombocytopenia (platelet count < 100
×109/L);

9. Treatment
 Individualized treatment

 Gestational hypertension: Rest, sedation, close monitoring of mother and infant,


Anti-hypertension according to the indication.

 Preeclampsia: sedation, antispasm, anti-hypertension according to the indication,


monitoring maternal and infant, terminate pregnancy timely.

 eclampsia : control the seizures , terminate pregnancy timely

10. Treatment
IV. Assessment monitoring: BP, assistant examination, NST

V. Common treatment: left lateral position, diet; enough protein


calories III. Antispasm: MgSo4

IV.
Seda
tion:
diaze
pam
V.
Antih
ypert
ensiv
e:

VIII. Diuretics

IX. Promote fetal lung maturation VIII. Terminate


pregnancy

11. Eclampsia
 Control seizure: first line medicine- MgSO4、 hibernate mixture

 Correct acidosis and hypoxia:4% NaHCO3

 Control hypertension

 Terminate pregnancy 2 hours after controlling seizure

Post-partum
 Prevent postpartum eclampsia: MgSO4, 24-48h after delivery.

 Monitoring BP and proteinuria 3-6d after delivery.

 Monitoring and record the hemorrhage

SouNok
DIABETES MELLITUS
1. Diabetes mellitus, a clinical syndrome character¬ized by deficiency of or
insensitivity to insulin and exposure of organs to chronic hyperglycemia, is the most

common medical complication of pregnancy 2. Types

I. DM (diabetes
mellitus prior to
pregnancy) II.
GDM( gestational
diabetes mellitus)

3. Diagnostic criteria
 There are three ways to diagnose preexisting dia¬betes mellitus and each way must
be confirmed by a follow up test. Criteria for diagnosing diabetes melli¬tus include:

 Symptoms of diabetes (polyuria, polydipsia, and/ or unexplained weight loss) plus a


casual plas¬ma glucose concentration ≥200 mg/dL.

 Fasting plasma glucose (at least 8 hours without eating) ≥126 mg/dL.

 Two-hour plasma glucose ≥ 200 mg/dL after drinking a 75 -gram glucose load.

GDM
Fasting glucose ≥ 5.1mmol/l ,1-hour ≥ 10 mmol/l,2-hour ≥ 8.5mmol/l
The diagnosis of GDM would be made with one out of the three values elevated

4. Complications
C. Maternal

 Preeclampsia

 Postpartum hemorrhage

 Ketoacidosis,

 Infection, Diabetic coma

D. Fetal

 Shoulder dystocia

 Abortion, intrauterine death

 Congenital anomalies

 Macrosomia,

 cardiomyopathy

SouNok
5. Treatment
 Antepartum care: diet and exercise.

 Insulin is added as needed for glucose control only after clear dietary errors are
noted and attempts at correction are done.

 Assessment of the fetus by glucose memory meters combined with


clinical/ultrasound assessment of fe¬tal growth cannot be replaced by other
antenatal tests. (fetal macrosomia, polyhydramnios-high risk )

6. Indications for early delivery include


I.

nonreassuri
ng fetal
testing, II.
Poor
glycemic
control.

V. worsening of uncontrolled hypertension,

VI. worsening renal disease, poor fetal growth

MEDICAL ILLNESS IN NORMAL PREG


Heart Disease in Pregnancy (the second leading cause of maternal death)
1. During the 32nd-34th gestational weeks, the blood volume increases to maximum (30-40
%↑)

• the burden of the heart is heaviest during delivery, the first stage is caused by
uterine contraction (blood enters the systemic circulation from the uterus)

• second stage when the breath is held: peripheral resistance ↑

• Third stage after delivery of the placenta, the blood volume increases suddenly.

Cardiovascular changes in normal pregnancy


 gestational period

 During delivery

 Puerperium

Returned blood volume increased in 3 days in postpartum and return to normal after 2~6
weeks.

SouNok
2. Classification of heart disease
I. Congenital heart disease

• Left-to-right shunt: atrial septal defect, ventricular septal defect, patent ductus
arteriosus.

• Right-to-left shunt

• Splitless : pulmonary stenosis, aortic stenosis, marfan syndrome

II.

Rheumatic
heart
disease 
rheumatic
valvular
lesion

• Mitral stenosis: the most common type.

• Pulmonary congestion, pulmonary edema.

Mild - tolerate pregnancy


Severe-surgery before pregnancy or terminate earlier

 mitral insufficiency: tolerate

 Aortic stenosis:surgery before pregnancy

 Aortic insufficiency:tolerate pregnancy

 The heart disease of hypertensive disorder

 Peripartum cardiomyopathy (PPCM) 


Myocarditis: after viral infection.

3. The influence on pregnancy


V. Incidence of Abortion, preterm delivery, FGR, fetal distress, neonatal
asphyxia,fetal death raised

VI. increased rate of cesarean-section

VII. toxic reaction of drug on fetus

VIII. inherited congenital heart disease(ventricular septal defect,Marfan's


syndrome)

4. Complications
e. heart failure: 32-34weeks, delivery stage, puerperium(3d)

f. Subacute infective endocarditis

g. Anoxia and cyanosis

h. venous embolism and pulmonary embolism

SouNok
5. NYHA Counselling
 Grade 1: uncompromised & no limitation on physical activity.

 Grade 2: slightly compromised/ slight physical activity limitation

 Grade 3: markedly compromised/ discomfort with less than ordinary activities

 Grade 4: severely compromised with discomfort even at rest

6. Pre-conception counseling
 Suitable for pregnancy

• If the damage to heart function is slight

• grade Ⅰ、Ⅱ

• without heart failure in the past

• without complications

 Not suitable for pregnancy

• the damage of heart function is serious

• grade Ⅲ 、Ⅳ

• congestive heart failure history

• serious cardiac arrhythmias,

• pulmonary hypertension

• age >35

6. The diagnosis of early heart failure


 Less than ordinary activity causes discomfort: palpitation, chest distress, short
breath.

 Heart rate > 110/minute and breath rate > 20/minute at rest.

 Orthopnea.

 Auscultation-a few moist rales appear and persistent at base of lung, cannot vanish
after coughing.

ANEMIA IN PREGNANCY
1. Gestational anemia: Hb <110g/L, Hct <0.33 caused mostly by hemodilution and
Iron deficiency. Mild:
Hb> 60g/L;

severe: Hb ≤

SouNok
60g/L The

effect of

anemia on

pregnancy

 Effects on mother:
 poor tolerance to surgery、childbirth and anesthesia

 maternal mortality rate ↑(anemic heart disease, preeclampsia,


placenta abruptio)  Puerperium infection rate ↑

 Effects on baby:
 Amount of iron transferred to the fetus is unaffected even if the mother suffers from
iron deficiency anemia. So the neonate does not suffer from anemia at birth.

 lack of folate→fetal neural tube defects

 FGR、preterm labor 、Fetal distress even dead

2. Treatment of IDA
IV. Supplementary iron therapy: oral + Vit C (aids absorption)

V. blood transfusion: Hb≤60g/L

VI. Intrapartum and postpartum treatment

• preventing postpartum hemorrhage

• preventing infection

4. Clinical manifestation of diagnosis


 The onset is usually insidious and is first revealed in the last trimester or may be
acutely manifested in early puerperium

 Anemia:Pallor of varying degree,fatigue、dizziness、short breath。

 symptom of digestive tract:loss of appetite、nauseas and vomit, Anorexia or


protracted vomiting ,Occasional diarrhea, ulceration in the mouth and tongue

 symptom of peripheral neuritis:hand and foot numbness, tingling (Vit B12)

 others:unexplained fever,enlarged liver and spleen

PREGNANCY WITH SURGICAL DISEASES


2. Pregnancy surgical diseases
 Acute appendicitis: Acute appendicitis is the most common extrauterine
complication of pregnancy for which laparotomy is performed. most cases occur in
the second and third trimesters

SouNok
 Acute cholesystitis and cholelithiasis, Acute intestinal obstruction

3. Symptoms of acute appendicitis


 Atypical, Right lower- or middle-quadrant pain; (vague pain )

 Rectal and vaginal tenderness are present in 80% of patients, particularly in early
pregnancy.

 Nausea, vomiting, and anorexia are usually present, as in the non-pregnant patient.

3. Signs
 Tenderness, Upward displacement of the appendix.

 After the first trimester, the appendix is gradually displaced above McBurney's point,
with horizontal rotation of its base. The migration continues until the eighth month
of gestation.

4. Differential diagnosis
 Pyelonephritis (the most common misdiagnosis)

 ruptured corpus luteum cyst, adnexal torsion, ectopic pregnancy

 abruptio placentae, early labor, round ligament syndrome

 chorioamnionitis, degenerating myoma, salpingitis, cholangitis

5. Symptoms of acute cholecystitis


• Biliary colic attacks are often of acute onset, seemingly triggered by meals, and
may last from a few minutes to several hours.

• anorexia, nausea, vomiting, dyspepsia , and intolerance of certain foods,


particularly those with high fat content ( nonpregnant state)

6. Signs
I. Fever, Right upper-quadrant pain, Murphy's sign (tenderness under the liver with deep
inspiration)

7. Differential Diagnosis
g. Appendicitis

h. symptoms of digestive track

i. pain, WBC ↑

j. AST, ALT, bilirubin and WBC ↑

k. Severe preeclampsia with associated right upper-quadrant abdominal pain and


abnormal liver function tests.

l. proteinuria, nondependent edema, hypertension

SouNok
.
8 Causes of acute intestinal obstruction
 Adhesion 60%

 Volvulus 25%

 Others 15%

Treatment of acute intestinal obstruction


 The same management as non-pregnant patients  The cornerstones of therapy
a. Bowel decompression

b. IV hydration

 timely surgery

NORMAL LABOUR

1. Labor is a sequence of uterine contractions that result in effacement and dilatation


of the cervix and voluntary bearing-down efforts leading to the expulsion per vagina of the
products of conception.

2. The essential factors of labour


A. The power of delivery:

From Uterine contraction


 Rhythmic (contraction-ascending, acme, descending: relaxation)

 Symmetrical,

 Polarity,

 Retraction

E. The passage (the pelvis)


F. The passenger (the fetus)
G. The psychic (the mother)

3. The birth canal;


 Bony pelvis:

 Pelvic planes and line:

 Pelvic inlet

Obstetric conjugate (11cm): from the sacral promontory to the pubic symphysis

SouNok
Transverse of pelvic inlet (13cm): left-right diameter

The obliques of pelvic inlet (12.7cm)

 Midpelvis

 Pelvic outlet

3. labour mechanism of occipital presentation


Engagement → Descent→ Flexion→ Internal rotation→ Extension→ Restitution&
External rotation→ Fetus delivery
4. Engagement
 In the primigravida: occurs late in pregnancy commonly in the last 2 weeks.

 In the multiparous patient: occurs with the onset of labor.

 Through the biparietal diameter : pelvic inlet plane→occipito frontal


diameter → bi-ischial diameter

5. Flexion: fetal presentation →to the floor of pelvis→Levator ani muscle


→flexion→occipito frontal diameter →suboccipito-bregmatic diameter (smallest)

6. Internal rotation: with the descent of head into the midpelvis, rota¬tion occurs.
The sagittal suture occupies the antero-posterior diameter. Internal rota¬tion normally
begins with the presenting part at the level of the ischial spines.
The levator ani muscles form a V-shaped sling that tends to rotate the occiput anteriorly.
In cases of occipito-anterior occipital, the head has to rotate 45 degrees, to pass beneath the
pubic arch.

7. Extension
Because the vaginal outlet is directed upward and forward, extension must occur before the
head can pass through it.
As the head continues its descent (Uterine and Levator ani muscle contraction) there
is a bulging of perineum .Fur¬ther extension follows extrusion of the head beyond the
introitus.

Ppt exercise obg gyne docx

OBG GYN PPT MCQS

SouNok
OVARIAN TUMORS
Risk factors for ovarian malignancy include all of the following except
which?
A.BRCA carrier
B.Multiparity
C.Older age(menopausal)
D.Lynch Ⅱ syndrome
Which ultrasound findings is concerning for ovarian malignancy?
A.Solid consistency
B.Unilocular
C.Thin-walled cyst
D.Smooth boders
E.Unilateral

AMENORRHEA
A 32-year-old G1P1001 woman presents to your office with the
chief
complaint of amenorrhea because her most recent vaginal delivery
1 year ago. She notes that she had an uncomplicated pregnancy,
followed by the delivery of a healthy baby boy. Her delivery was
complicated by an intraamniotic infection as well as a postpartum
hemorrhage requiring a postpartum dilation and curettage (D&C).
After her delivery, she breastfed for 6 months, and during this time,
she had scant and irregular vaginal bleeding. After stopping
breastfeeding 6 months ago, she notes the absence of menses,
but instead has monthly painful cramping, which seems to be
getting worse. She remarks that prior to her pregnancy, she had
normal, regular menses, which were not too heavy or painful. She
and her husband would like to have another child, and have been
having unprotected intercourse for the last 6 months without
achieving a pregnancy. Your review of systems is otherwise
negative. You perform a physical examination, which is normal
other than a slightly enlarged,tender uterus. A urine pregnancy test
in the office is negative.

1. What is the most likely diagnosis?

SouNok
A.Sheehan syndrome
B.lactational amenorrhea
C.Asherman syndrome
D.Primary ovarian insufficiency (POI)

2. You suspect Asherman syndrome and perform a


hysterosalpingogram, which reveals multiple synechiae within the
uterus, confirming your suspicions. Your next step in therapy is
which of the following?

A.Diagnostic and operative hysteroscopy


B.Provera 10 mg for 5 days in attempt to achieve a withdrawal bleed
C.IVF
D.Place an intrauterine device (IUD)
E.Inform your patient that unfortunately, she is ‘barren’ and will not be
able to carry a pregnancy again

3. What obstetrical complication might you be concerned about for


your patient in her next pregnancy?

A.Preterm labor
B.Cervical insufficiency
C.Preeclampsia
D.Placenta accereta

GESTATIONAL TROPHOBLASTIC DISEASES

1.Which is the first choice of the treatment for choriocarcinoma best?


A. hysterectomy
B. Chinese medicine
C. immunotherapy
D. chemotherapy
E. radiation therapy
2.How to differentiate invasive mole with CC?

SouNok
A seen villous pattern, is CC
B. with lung metastasis, is CC
C. followed by mole, is CC
D. followed by normal labor or abortion, is CC
E. none of them

3.Which is the most helpful examination for diagnosis of HM?


A. Ultrasound
B. CT
C. HCG
D. pelvic examination
E. Hysteroscopes
4.Which of the following choice is the effective and suitable management
for HM?
A. chemotherapy
B. hysterectomy
C. radiotherapy
D. evacuation
E. Following observation

5.A woman who has HM most often presents with which of the following
symptoms?
A. Uterine bleeding post amenorrhea
B. Abdominal pain
C. postmenopausal bleeding
D. vaginal discharge
E. infection

A 17-year-old G1 P0 patient presents to your office with vaginal bleeding


at approximately 8 weeks’ gestation by her last menstrual period. Her
examination is benign with a 10-week-sized uterus, a closed cervical os,
and a small amount of blood within the vaginal vault. You order a
complete pelvic ultrasound that shows an intrauterine gestational sac
containing a fetus measuring approximately 6 weeks’ gestation. Doppler
sonography is unable to demonstrate any fetal heartbeat. The placenta
demonstrates marked thickening and increased echogenicity with
scattered cystic spaces within the placenta. A serum β-hCG is 52,000
mIU/mL.

SouNok
1.What is the most likely diagnosis?
A.Complete molar pregnancy
B.Incomplete molar pregnancy
C.Incomplete abortion
D.Missed abortion
E.Inevitable abortion

2.You decide to perform a suction D&C. When giving informed consent,


you discuss the risk most commonly encountered in this operation. Which
of the following is the most common risk associated with suction D&C?
A.Uterine perforation
B.Infection
C.Damage to the bladder
D.Uterovaginal bleeding
E.Need for future surgery.

3.After pathology returns, you discuss the findings with your patient in
follow-up at your office. Which of the following is most accurate when
discussing risk of persistent gestational trophoblastic disease (GTD)?
A.2% to 4%
B.Less than 1%
C.6% to 10%
D.11% to 15%
E.Greater than 20%.

ANATOMY OF THE FEMALE REPRODUCTIVE SYSTEM

1、Which part is prone to form hematoma if injured ?


A: mons pubis
B: labium majus
C: labium minus
D: clitoris
2、which is the lowest part of pelvic?

SouNok
A: Rectouterine pouch
B: Utero vesical pouch
C: perineum
D: cerxix

3、Which is not belong to vaginal vestibule?


A: vestibular bulb
B: major vestibular glands
C: clitoris
D: vaginal orifice
4、Which is not the ligaments of uterus?
A: round ligament
B : broad ligament
C : cardinal ligament
D: Sacrospinous ligament

5、Which ligament keeps the anteversion of the uterus?


A: round ligament
B: broad ligament
C: cardinal ligament
D: Sacrospinous ligament
6、Which is not right about uterus?
A: it is consist by corpus ,Isthmus and Cervix.
B: the size is about 2-3cm *4-5cm*7-8cm
C: Isthmus is a constricted part situated between the body and the
cervix.
D: the muscles of uterus arranged in the same direction.

SouNok
7、Which ligament draw the cervix backward and upward?
A: round ligament
B: broad ligament
C: cardinal ligament
D: uterosacral ligament
8、Which is not right about ovary?
A: they are paired sex glands, the size is about 4 cm* 3 cm *1 cm
B: it is attached to the pelvic by infundibulopelvic ligament
C: The ovary is consists of outer cortex and inner medulla. Medulla is the
main part of ovary.
D:The ovary is covered by germinal epithelium. without peritoneum.

9、Which is not right about oviduct?


A: Isthmus portion is the most narrow part
B: the fertilization occurs in the ampulla portion
C: it is devided into 4 parts
D: it can transport of the gametes, to facilitate fertilization and survival
of zygote through its secretion

CASE (GIVEN):

SouNok
ANS:
1) What is the most probable diagnosis of the patient?
The most probable diagnosis of the patient is Uterine Leiomyoma.
(Fibroids)

Differential Diagnosis of the patient are:-

 Adenomyosis
 Gravid uterus
 Ovarian tumor
 Pelvic inflammatory mass and uterine malformation.

2) What is the basis of my diagnosis?


The basis of my diagnosis is based on the following factors:
Symptoms:
I. Prolonged heavy menses.
II. Premenstrual spotting.
Which are associated with Abnormal uterine bleeding, the most
common and most important clinical manifestation of myoma.
Signs: Associated with the size, location, number and whether there is

SouNok
degeneration.
 Gynecological examination.
Showed a normal Cervix, enlarged uterus with a nodule about
7cm felt in the anterior wall of the uterus.
 Color Ultrasonography showed low echo nodule seen in the
myometrium of the uterus about 7~6 cm in size with clear
boundaries.
Bimanual examination, hysteroscopic examination, laparoscopic
examination and MRI can be also done to confirm the diagnosis.

3) How to treat?
In general the choice of treatment depends on
1. Symptoms
2. Patient`s age
3. Pregnancy status, desire for future pregnancies
4. General health
5. The size, location, and state of leiomyomas.
The scope of treatment for the above mentioned patient:
 Expectant management:
--most cases of uterine fibroids do not require treatment, and
expectant management is appropriate. If there is no symptom,
myoma is small or if the patient is postmenopausal.
---follow-up every 3-6 months to monitor the size and growth.
 Supportive management.
 Treat anemia with iron supplements.
 Medical therapy.
 GnRH agonists: can shrink fibroids and decrease
bleeding by decreasing circulating estrogen levels.
 The tumors usually resume growth after the medications
are discontinued.
 temporizing measure for women nearing

SouNok
menopause;shrink fibroid size prior to surgical treatment
of uterine fibroids.
 Surgical treatment (if indications are present):
The choice of surgery
Myomectomy: wish to preserve their fertility, myoma
resection. Myomectomy should be planned for the
symptomatic patient who wishes to preserve fertility or
conserve the uterus. As our patient has desire for future
pregnancies, this is the only choice of surgical treatment for
her.

 Abdominally.
 Laparoscopically.
 Hysteroscopically.
 Vaginally.

1. What are the complications of IUDS?


Ans: Complications of IUDS:
1) Perforation of the uterus and IUD ectopia
2) Infection
3) Incarceration
4)Displacement or expulsion of IUD
5)Pregnancy with IUD in place
6)IUD and ectopic pregnancy

2. State the management of Acute Fetal Distress.


Ans: Management of Acute Fetal Distress:
1. Fetal oxygen should be improve as soon as possible

2. General treatment:

SouNok
Lying on the left lateral position, oxygen inhalation etc.

3. Etiological treatment:
Termination of pregnancy as soon as possible

1.How many categories of ovarian cancer are divided into? And what
they are?

Ans: According to WHO classification, ovarian cancer is divided into four


histological categories.
They are:
Epithelial cell tumors
Germ cell tumors
Sex cord-stromal tumors
Metastatic Tumors

These four categories are briefly described below:

1.Common "epithelial" tumors (may be benign, borderline, or


malignant)
 Serous tumors
 Mucinous tumors
 Endometrioid carcinoma
 Clear cell adenocarcinoma
 Brenner tumor
 Mixed epithelial tumors
 Undifferentiated carcinoma

 50%-70% of all ovarian tumors.

 More than 85%-90% of all malignant ovarian tumors.

SouNok
 20+year, 2/3>55 years

2.Sex cord stromal tumors Granulosa-stromal cell (Granulosa


cell \ Thecoma-fibroma)
 Sertoli-Leydig cell tumor (androblastoma) Gynandroblastoma

 4.3%-6%

 Granulosa-theca cell tumor--"functioning tumor".

 0-25+year

3. Germ cell tumors

 Teratoma (Immature /Mature )

 Dysgerminoma

 Endodermal sinus tumors

 Embryonal carcinoma

 Polyembryoma

 Choriocarcinoma

 Mixed forms 20%-40%

 Teratoma -the most common.

 All ages

SouNok
4. Secondary (metastatic) tumors

o 5%-10% all ovarian tumor

o 25%of all ovarian malignancies.

o usually from the gastrointestinal tract, known as Krukenberg


tumors

GYNE MCQ docx

一、Choice questions(Choose the best answer for each question,1


point for each question )
1、Pelvis is composed of:
a.Hip, sacrum, coccyx composition;
b.Ilium, ischium, pubis composition

SouNok
c.Ilium, sacrum, coccyx composition;
d.Hip, pubis, sacrum composition;
e.Pubis, sacrum, coccyx form
2、If a women's menstrual cycle is 35 days, which day ovulation
probably occurred in menstrual cycle?
a. Day 14
b. Day 21
c. Day 11
d. Day 31
e. Day 17
3、Which is the most common fetal presentation ?
a、Cephalic presentation
b、Breech presentation
c、Shoulder presentation
d、Face presentation
e、compound presentation
4、The scope of the normal fetal heart rate is?
a.100-120bpm
b.110-120bpm
c.120-150bpm
d.120-160bpm
e.120-180bpm
5、Which is the most common sign used to describe the extent of fetal
head descending?
a. sacral promontory
b. ischial spine
c. ischial tuberosity

SouNok
d. cervix
e. coccyx
6、The most common cause of postpartum hemorrhage is?
a. cervical lacerations
b. placenta retention;
c. Placental adhesion
d. uterine atony
e. coagulation defect
7、Normal value of Biparietal diameter (BPD)is :
a.10.0 cm
b.8.8 cm
c.9.1 cm
d. 9.5cm
e.9.8 cm
8、Prolonged second stage for primipara is the second stage over:
a. 1h
b. 2h
c. 3h
d. 2.5h
e. 3.5h
9、Which is the content of birth canal except?
a.abdomen
b. pelvis
c. lower uterine segment
d.cervix
e.vagina

SouNok
10、Which is commonly found in the narrow of pelvic inlet plane?
a. gynecoid pelvis
b. flat pelvis
c. funnel-shaped pelvis
d. anthropoid pelvis
e. deformed pelvis
11、Umbilical cord has:
a、one umbilical vain and two umbilical arteries
b、two umbilical vain and one umbilical arteries
c、two umbilical vain and two umbilical arteries
d、one umbilical vain and one umbilical arteries
e、three umbilical vain and one umbilical arteries
12、Which of the following is not a feature of uterine contractility:
a. regularity
b. symmetry
c. intermittent
d. polarity
e. retraction
13、Which of the following is not the clinical manifestation of the first
stage of labor?
a.regular uterine contraction
b. rupture of fetal membrane
c. dilatation of cervix
d. delivery of placenta
e. descending of fetal presentation
14、The most fundamental pathophysiologic change of hypertensive
states of pregnancy is :

SouNok
a. imbalance of prostaglandin and thromboxane A
b.generalized vasospasm
c. excessive retention of sodium and water
d. blood concentration
e. hypercoagulable state
15、The main treatment of ovarian cancer is :
a. drug therapy
b. radiotherapy
c. chemotherapy
d. surgery + chemotherapy
e. biological treatment
16、The diagnosis of early cervical cancer is based on ?
a. occasional bloody vaginal bleeding after coitus
b. colposcopy
c. pelvic examination
d. pap smear lf cervix
e.biopsy of cervix
17、The most common complication of ovarian tumor is:
a. intracystic hemorrhage
b. become malignant
c. rupture
d. torsion
e. infection
18、Menstrual blood contains :
a. blood;
b. endometrial debris

SouNok
c. cervical mucus
d. vaginal exfoliated cells
e. all above
19、Infertility associated with dysmenorrhea frequently occurs in:
a. polycystic ovary syndrome
b. endometriosis
c. endometritis;
d. ovarian cysts
e. myoma of uterus
20、Bleeding of placenta previa early more common found in:
a. complete placenta previa
b. partial placenta previa
c. marginal placenta previa
d. marginal and partial placenta previa
e. complete and partial placenta previa
21、Which is the most common symptom of uterine myoma:
a. abnormal uterine bleeding
b. urinary retention
c. constipation
d. infertility
e. dysmenorrhea
22、The most common site of tubal pregnancy is :
a. umbrella department
b. isthmus
c. ampulla
d. interstitial

SouNok
e. cervical diverticulum
23、There are a large number of purulent frothy yellow-green vaginal
secretion, the most common disease is:
a. bacterial vaginosis
b. candida vaginitis
c. trichomonas vaginitis
d. cervical erosion
e. senile vaginitis
24、The most common reason of ectopic pregnancy is :
a、tubal inflammatory disease
b、IUD
c、passage of the fertilized ovum
d、smoking
e、drinking
25、Pelvic floor has:
a. 1 layer;
b. 2 layers;
c. 3 layers;
d .4 layers;
e. 5 layers;
26、Which of the following does not belong to the soft birth canal:
a. lower uterine segment
b. cervix
c. pelvic axis
d. vagina
e. pelvic soft-tissue

SouNok
27、The normal average value of bispinous diameter is :
a. 8cm
b. 9cm
c. 10cm
d. 11cm
e. 12cm
28、Prolonged labor is the total stage of labor over:
a. 20h
b. 24h
c. 28h
d. 22h
e. 26h
29、Which fetal position is most harmful to mother and fetus?
a. occiput posterior position
b.transverse lie
c.breech presentation
d.compound presentation
e.occiput transverse
30、The most fundamental pathophysiologic change of hypertensive
states of pregnancy is :
a. imbalance of prostaglandin and thromboxane A
b.generalized vasospasm
c. excessive retention of sodium and water
d. blood concentration
e. hypercoagulable state
31、An 18 –year- old nulliparity presents at 36 weeks gestation feeling
slightly nauseated. Her blood pressure is 165/105 mm Hg and dipstick

SouNok
testing shows proteinuria of ++. She admits to no history of renal
problems now or in the past.Possible diagnosis is:
a. primary hypertension
b. eclampsia
c. chronic hypertension with superimposed preeclampsia
d.Severe Preeclampsia
e. glomerular nephritis
32、Partus maturus is :
a. Pregnancy over 28 weeks and under 37 weeks
b. pregnancy over 37 weeks and under 42 weeks
c. pregnancy over 37 weeks and under 40 weeks
d. pregnancy over 28 weeks and under 40 weeks
e. pregnancy over 28 weeks and under 38 weeks
33、A young woman, suddenly choking, difficulty breathing, pale, blood
pressure decreased during childbirth, the most likely diagnosis is :
a. postpartum hemorrhage
b. amniotic fluid embolism
c. placental abruption
d. uterine rupture
e. eclampsia
34、Which of the following has nothing to do with the pathogenesis of
amniotic fluid embolism?
a.hypertonic uterine dysfunction
b. premature rupture of fetal membranes
c. uterine rupture
d. cesarean section
e.hypotonic uterine dysfunction

SouNok
35、Which of the following is not the cause of postpartum hemorrhage:
a.Utrine atony
b. Fetal distress
c.Obstetric lacerations
d.Retained placental tissue
e.Coagulation defects
36、When will pathological retraction ring occur in the clinical?
a. fetal malformation
b. uterine contraction fatigue
c. threatened rupture of uterine
d. soft birth canal anomaly
e.breech presentation
e. use antibiotic combined
37、The preferred treatment of postpartum hemorrhage caused by
uterine contraction fatigue is:
a. uterotonic agents
b.bimanual compression and massage
c.hysterectomy
d.pressure occlusion of the aorta
e. uterine packing
38、The most common amenorrhea is:
a. Uterine amenorrhea
b. ovarian amenorrhea
c. hypothalamic amenorrhea
d. pituitary amenorrhea
e. primary amenorrhea

SouNok
39、The most important pathogenic factor of Endometrial cancer is:
a. Long-term estrogen stimulation
b. obesity
c. hypertension
d. diabetes mellitus
e. genetic
40、The major metastasis route of corpus carcinoma is:
a. Lymphatic metastasis
b. direct diffusion
c. hematogenous metastasis
d. planting
e. None of the above
41、The most common site of metastases of choriocarcinoma is:
a. vagina
b. brain;
c. lung
d. liver and spleen
e. kidney
42、Which of the following belonging to benign ovarian tumor?
a. endodermal sinus tumor
b. theca cell tumor
c. dysgerminoma
d. granular cell tumor
e. Krukenberg tumor
43、The most common complication of ovarian tumor is:
a. intracystic hemorrhage

SouNok
b. become malignant
c. rupture
d. torsion
e. infection
44、The most common location of Endometriosis is :
a. ovary
b. fallopian tubes
c. the posterior wall of the uterus
d. uterosacral ligament
e. cul-de-sac
45、Which is the most common symptom of uterine myoma:
a.abnormal uterine bleeding
b. urinary retention
c. constipation
d. infertility
e. dysmenorrhea

Fill in the Blank questions


1、Mechannism of labor in vertex presentation consists of
Engagement、_____、______、______、_____、 Restitution
and_____、_____。(Descent,Flexion ,Internal
rotation ,Extension,External rotation ,Fetus delivery)
2、Complications of ovarian tumor
include_______、______、_______、_______。(torsion ,
rupture,infection,become malignant)
3、Internal genitalia includes_______、______、_______、_______。
(vagina, uterus, oviducts, ovaries)
4、Classification of placenta previa includes_____ 、_____、_____。

SouNok
(Complete placenta previa, Partial placenta previa, Marginal placenta
previa)
5. Clinical classification of Spontaneous abortion includes ( Inevitable
abortion、Missed abortion、Septic abortion .Threatened abortion,
Incomplete abortion, Complete abortion, Habitual abortion)
1.Which is the most common complication during vaginal
delivery is a diabetic women?
a. Uterine inertia
b. Shoulder dystocia
C. PPH
d. Excessive moulding of head

2. Late hyperglycemia in pregnancy is associated with:


a. Macrosomia
b. IUGR
C. Postmaturity
d. Congenital malformation

3.Cardiac failure is most likely in pregnancy at:


a. 32 week
b. 1st stage of labor
C.3rd stage of labor
d. 4th stage of labor

4.Nulliparity,28 year old,36 weeks gestation,having a Lower


extremity edema for half a month,Having a headache for
three days,this morning feel blurred vision,and severe
headache.urine dipstick testing shows proteinuria of ++.She
admits to no history of renal problems now or in the past.

SouNok
1.Which is most likely to be found by Physical examination?
A.HR>110bpm
B.BP 160/110mmHg
C.splenomegaly
D.hepatomegaly
2.If BP 145/98mmHg,Possible diagnosis?
A.Gestational hypertension
B.mild preeclampsia
C.chronic hypertension with superimposed preeclampsia
D.Severe Preeclampsia
5.Nulliparity,24 year old,33 weeks gestation,Having a
headache for 6 days.BP 180/120mmHP96bpm,breech
presentation,FHR150bpm,severe edema.
1.the most important assist examination?
A.Red blood cell count and hemoglobin measurement
B.Blood sedimentation
C.fundus inspection
D.urinalysis
2.Which treatment should not be given right now?
A.rest ,left lateral position
B.cardiac stimulant
C.diuresis
D.Anti-hypertension
E.anti-spasm
3.Multiple organs are affected,except?
A.brain B.eye C.lung D.heart E.kindy

SouNok
6.Nulliparity,22 year old,37 weeks gestation,feels dizzy and
proceeds to have a tonic—clonic seizure,lasting 1minutes.BP
180/105mmHg,edema,LOA,FHR142bpm,Irregular
contractions.admit to hospital in emergency.
1.Which is the first choice for diagnosis?
A.Immediate urethral catheterization,analysis the urine protein.
B.RBC and PLT
C.Blood gas analysis
D.24-hour urinary protein quantity
2.The preferred treatment should be?
A.Diazepam,10mg,im
B.25%MgSO410mL iv(15-20min)
C.diuresis
D.hibernate mixture,1/2,im
3. In the process of lift patients,the patient convulse again.Which
treatment is not correct?
A.Cesarean section at once
B.hibernate mixture
D.inhale the oxygen
E.keep in a quiet and dark room
4.Which of the following situation is the most dangerous?
A.BP 165/105 mmHg,
B. Proteinuria 4g/24h
C. FHR142bpm
D.persistent abdominal pain,tenderness on the uterus,Vaginal
bleeding,BP120/90 mmHg
E.Somnolence
5.Her abdominal pain was aggravated,BP105/75 mmHg,urine

SouNok
volume 600mL/24h,the next step is
A. 25%MgSO 60mL , iv,
B. hibernate mixture
D. diuresis
E. Cesarean section at once
7.A 37 year old primigravida attends the prenatal clinic for booking
at 9 weeks gestation. She tells you she has a history of high blood
pressure, which has been investigated by physicians in the past
with no cause found. Her blood pressure is usually Controlled with
atenolol 100mg daily.(atenolol-antihypertensive drug)
Possible diagnosis?
i] Gestational hypertension ii] Eclampsia
iii] Essential hypertension iv] Preeclampsia
8.An 18 year old nulliparity presents at 36 weeks gestation
feeling slightly nauseated.Her blood pressure is 165/105 and
dipstick testing shows proteinuria of ++.She admits to no
history of renal problems now or in the past.
Possible diagnosis?
i] Gestational hypertension ii] Eclampsia
iii] chronic hypertension with superimposed preeclampsia
iv]Severe Preeclampsia
9.Two days after a normal delivery of her second baby,a
mother feels nauseous and unwell and proceeds to have a
tonic-- clonic seizure.
Possible diagnosis:-
i] Gestational hypertension ii] Eclampsia
iii] Essential hypertension iv] Preeclampsia
10.A 25 year old nulliparous patient is found to have a
diastolic blood pressure of 95 mmHg at 32 weeks of

SouNok
gestation. Her blood pressure recording at 10 weeks of
gestation was 115/78 mmHg
Possible diagnosis?
i] Gestational hypertension ii] Eclampsia
iii] Essential hypertension iv] Preeclampsia
11.Which is irrelevant to preeclampsia?
A.multifetation
B.polyhydramnios
C.nulliparity
D.maternal age below 20 or over 35 years
E.Placenta previa
12.In patients with severe pre-eclampsia,blood pressure should
be greater than or equal to?
A.140/100mmHg
B.150/90mmHg
C.150/100mmHg
D.160/100mmHg
E.160/110mmHg
13.Application of magnesium sulfate treatment of preeclampsia
during pregnancy, breathing should not be less than a minute?
A.12 次/分
B.16 次/分
C.18 次/分
D.20 次/分
E.22 次/分
14.Nulliparity, 26 years old.38 weeks of pregnancy, medium-
term prenatal normal.36 weeks pregnant when you feel a

SouNok
headache,vertigo.Examination:blood pressure
160/110mmHg,urine protein (++),irregular contractions,
fetal heart rate 134 beats / min.
The most appropriate treatment should be?
A. Follow-up outpatient treatment
B. Intravenous infusion of magnesium sulfate
C. Warm soapy water enema labor induction
D. Cesarean section
15.Nulliparity, 27-year-old , 37weeks’gestation, the basis of
blood pressure is not high. 5 days ago ,she felt headache and
blurred vision, checking blood pressure 160/100mmHg, urine
protein (++), fetal heart rate 148 beats / min, urinary
estrogen / creatinine was 11.
How to deal with the most appropriate?
A.Active treatment to 39 weeks of termination of pregnancy
B.Active treatment 24-48 hours after the termination of pregnancy
C.Active treatment, waiting for a natural childbirth
D.Intravenous infusion of oxytocin
E.An immediate Cesarean Section
16.Female,28 years old,has married two years but not
pregnant;Menstruation regular,dysmenorrhea five years,
the man body health. Bimannual examination: the size of
the uterus is normal,and inactive, tenderness,bilateral
adnexal can touch 6 cm cyst;BBT is biphasic
pattern,according to the above symptoms and
signs,considering the causes of infertility is
A.PID
B.TB pelvic inflammation
C.Edometriosis
D: Ovarian cancer
E: Hydrosalpinx

SouNok
17.Female,36years,8 years after abortion,Now 3 months after
menopause,vaginal bleeding for three days,uters is enlarged but
smaller than gestational date, β-HCG>100kIU/ L
the most possible diagnosis?
A.Threatened abortion
B.Ectopic pregnancy
C. Molar pregnancy
D. invasive mole
E.Choriconoma
18.Female,42 years,Hydatidiform mole,The size of uterine
body>G14w
the best treatment is ?
A. Suction &curettage:
B. hysterectomy directly
C.Suction first and then hysterectomy
D. Prophylactic chemotherapy first ,and Suction
19.What is the commonest cause of Spontaneous abortion?
Abnormalities of chromosomes
20.Which of the following has nothing to do with spontaneous
abortion?
a.inadequate luteal function
b.severe cervical laceration
c.poorly controlled diabetes mellitus
d.uterine malformations
e.Multipara
21.Which of the following prone to hemorrhagic shock easily?
a.threatened abortion
b.inevitable abortion
c.incomplete abortion
d.complete abortion
e.missed abortion
22.The most common site of tubal pregnancy is?
a. umbrella department
b. isthmus

SouNok
c. ampulla
d. interstitial
e. cervical diverticulum
23.The most common reason of ectopic pregnancy is?
a.tubal inflammatory disease
b.IUD
c.passage of the fertilized ovum
d.smoking
e.drinking
24.The clinical symptom of uterine myoma is most greatly
associated with which character of it?
A.the tumor size
B. the number
C.the situation
D.the age of patient
E.whether childbearing or not
25.Which of the following change would be seen mostly in myoma
of uterus combined with pregnancy?
A.hyaline degeneration
B.cystic degeneration
C.red degeneration
D.sarcomatous change
E.calificated degeneration
26.Which of the following belong to uterine amenorrhea?
A. Asherman syndrome
B. Sheehan syndrome
C. Turner syndrome
D. premature ovarian failure
E. polycystic ovary syndrome
27.Which of the following is the diagnosis of hydatidiform

SouNok
mole?
A:Uterine is larger then normal, without fetal hrart rate.
B:Irregular vaginal bleeding after menopause
C:grape-like organization in Vaginal effluent
D:Severe nausea and vomit in early pregnancy,with
Preeclampsia
28.Which of the following is the most reliable way of diagnosing
hydatidiform mole?
A.β-HCG
B.X-ray
C.ultrosound
D.CT
29.Which is the best way of contraception for hydatidiform mole
patients?
A.IUD
B.Oral medicine of contraception
C.Injectable medicine of contraception
D.Tools for contraception, such as condom, vagina diaphragm

30.Which is not the pathological changes of hydatidiform mole?


A. Trophoblast proliferate

B.hemorrhage in the cavity

C:Edema of villous stroma

D:No embryogenetic blood vessels in stroma

31.which is not suitable for Suction &curettage of hydatidiform


mole?

A:once confirmed, Suction &curettage timely

SouNok
B:prepare blood for transfusion

C:try one’s best to clean the tissus at one time

D:Tissue from the decidua basalis for pathologic study

32.Which of the following is false about endometriosis?


A. commonly occurs in women of reproductive age
B. the symptom may not palliate or disappear after pregnancy
C. the most common location is ovaries
D. adenomyoma is a kind of endometriosis
E. the endometrium of abnormal growths occur very rarely
malignant change
33.A woman who has endometriosis most often presents with
which of the following symptoms?
A. Secondary dysmenorrhea that worsens over time
B. Abdominal pain
C. postmenopausal bleeding
D. vaginal discharge
E. Infection
34.Which is the most helpful examination for diagnosis of
adenomyosis?
A.Ultrasound
B.CT
C.HCG
D.pelvic examination
E.Hysteroscopes

35.Treatment of endometriosis is no use usually


A. Medical treatment
B. laparoscopes

SouNok
C. chemotherapy
D. surgical operation of Fertility preservation
E. Radical surgery
36.The most common site in the endometriosis lesions is
A. myometrium
B. ovary
C. cervix
D. douglas' pouch

E. uterosacral ligament

SouNok

You might also like

pFad - Phonifier reborn

Pfad - The Proxy pFad of © 2024 Garber Painting. All rights reserved.

Note: This service is not intended for secure transactions such as banking, social media, email, or purchasing. Use at your own risk. We assume no liability whatsoever for broken pages.


Alternative Proxies:

Alternative Proxy

pFad Proxy

pFad v3 Proxy

pFad v4 Proxy